Clinical Social Work Licensure Exam practice

Réussis tes devoirs et examens dès maintenant avec Quizwiz!

clozapine (clorazil)

bipolar meds

At age six the child will begin to have their baby teeth start being replaced by their permanent teeth.

...

Normal aging changes: - eye sight loss - hearing loss - taste buds change - touch and small sensitivity - arteries stiffen - bladder problems, frequent urination - body fat shifts to surround deeper organs - bones lose minerals - brain lose some of the structures that connect nerve cells - kidneys shrink/less efficient - lung tissue loses elasticity - muscle mass decline - skin and nails grow slowly

...

Non-verbal forms of communication

body language, dress, space, tone, some which is unconscious

2. You have just had your first session with a 24-year-old college student. She is seeing you following the break-up of a two-year relationship, which occurred without warning about six weeks prior to this visit. As she explained it, "He met someone else and just moved on." She has been having trouble sleeping and concentrating on her studies since that time. Today she presents as dysphoric and tearful, but is affectively expressive and responsive to humor and other interactive stimuli. The university she attends is a considerable distance from her family and friends, leaving her with limited support during this difficult time. The most appropriate diagnosis would be: a. Primary insomnia. b. Major depression. c. Adjustment disorder with depressed mood. d. Acute stress disorder.

2 - C: Adjustment disorder with depressed mood. Criteria for this disorder includes a time-limited nature, usually beginning within three months of the stressful event, and lessening within six months-either with removal of the stressor or through new adaptation skills. Adjustment disorder is a "sub-threshold disorder," allowing for early classification of a temporary condition when the clinical picture remains vague. While the patient does have insomnia, it arises from the stressful loss and not as an independent condition. Many of the essential criteria for a major depression are absent (weight loss, psychomotor agitation, blunted affect, etc), although without successful treatment this condition could emerge. The diagnosis of acute stress disorder is not appropriate as the precipitating event did not involve threatened or actual serious injury or death.

A young father of 2 girls comes to a mental health clinic shortly after the death of his wife. He complains that he is usually energetic and optimistic, but since his wife's death he feels depressed and unmotivated. Now he is having problems at work, completing his job tasks. He is a single father with all child rearing activities that are taking up a lot of his time. 9. The wife's death is viewed as: A) The presenting problem B) A precipitating stress C) A tragedy D) Irrelevant to his functioning

A precipitating stress

5. A social worker who uses behavioral techniques would be least interested in A) Operant conditioning B) A social work relationship C) Contract D) Model presentation

A social work relationship

All of the following are examples of primary prevention strategies EXCEPT: A. A social skills group for children with ADHD. B. Free informational sessions on immunizations for parents. C. Ongoing skills training for teens on how to build healthy relationships. D. Providing information on infant development to new parents.

A. A social skills group for children with ADHD. RATIONALE: A is the best answer because it does not represent a primary prevention strategy; the intervention is aimed at helping a targeted group of children manage a long-term problem, making it an example of tertiary intervention.

A white social worker at a hospital is asked to evaluate a black woman who was recently brought in by the paramedics. When the social worker approaches the client, she refuses to tell the social worker what happened and states repeatedly that she fears anything she says to the social worker will be used against her. The woman requests to instead speak with the Latina nurse who has been caring for her since arrival. What is the MOST likely explanation for the woman's behavior? A. Cultural paranoia, which represents a healthy reaction to racism. B. An episode of acting out of Borderline Personality dynamics. C. Pathological paranoia regarding all social workers. D. Resistance to assessment and treatment.

A. Cultural paranoia, which represents a healthy reaction to racism. RATIONALE: Answer A is the best answer because it takes into account the cultural issues presented in the case. B and C both pathologize the client's behavior; D makes a negative judgment about the client's behavior, when it may in fact be protective.

A social worker in a small community is contacted by a prospective client regarding therapy for conflicts she is experiencing at her job. The social worker knows that the client works at the agency where the social worker was previously a supervisor, although the social worker did not supervise the client. The prospective client insists that the social worker's knowledge of the agency environment and culture will benefit treatment because of increased understanding. What is the social worker's MOST appropriate response? A. Decline to provide therapy due to knowledge of the previous business relationship. B. Assure the client of confidentiality before beginning treatment. C. Provide treatment while maintaining self-awareness regarding boundary issues. D. Explore the meaning of dual relationships before beginning treatment.

A. Decline to provide therapy due to knowledge of the previous business relationship. RATIONALE: Answer A and D are your best options for this questions. A is a little stronger because in this case it would be better to refer her to someone not connected to the previous working environment. Providing a safe space for clients is an essential part of therapy. A client like this may assume that you will agree with her view of the situation but your previous knowledge of the environment could actually be a hindrance to your work together.

A social worker in private practice meets with a 90-year-old man who recently lost his wife of 60 years. The client reports that since his wife passed away, he has being feeling irritated, does not have much energy, and has trouble falling asleep. A hospital social worker met with the client and conducted a clinical evaluation for depression. What should the social worker do FIRST? A. Demonstrate acceptance of the client's feelings of loss. B. Provide psychoeducation about depression. C. Complete a mental status examination. D. Contact the hospital social worker for additional information.

A. Demonstrate acceptance of the client's feelings of loss. Rationale: A is the best answer because the client has experienced a significant loss and acknowledging and accepting his feelings is an effective way to help him feel more comfortable and encourage him to discuss his feelings and symptoms. B, C and D might all be useful interventions, but not before the social worker has acknowledged the client's feelings of loss regarding his wife.

A social worker receives a referral from parents regarding their 9 year-old daughter. A social history reveals symptoms consistent with a diagnosis of Major Depressive Disorder. What should the social worker do FIRST? A. Determine if the child has had a recent physical examination to rule out possible physical causes for her symptoms B. Under the diagnosis of Major Depressive Disorder, refer the child to a psychiatrist for medication C. Have the parents fill out a questionnaire to confirm the diagnosis of Major Depressive Disorder D. Begin cognitive behavioral therapy with the child to decrease symptoms

A. Determine if the child has had a recent physical examination to rule out possible physical causes for her symptoms RATIONALE: While B, C and D may all be appropriate at some time in the near future, option A is the best choice as medical conditions should always be ruled out before giving a diagnosis.

A client who is diagnosed with trichotillomania meets with a social worker after being referred by her primary care physician. The client says she feels guilty and ashamed, but is having difficulty stopping the behavior. She says that she doesn't even know why she started the behavior in the first place. What should the social worker do FIRST? A. Explore the feelings behind the behavior. B. Have the client wear a wig until she can stop the behavior. C. Reinforce that hair-pulling will likely lead to complete baldness. D. Teach the client thought stopping strategies.

A. Explore the feelings behind the behavior. Rationale: A is the best answer because understanding the psychological purpose of the behavior will best help the client understand and change the behavior. B, C, and D might all be part of the complete intervention strategy, but A is the best place to start in terms of addressing the behavior.

A 23-year-old woman is referred by her MD for therapy for anxiety. During the session, the client reports that she was sexually molested by her gymnastics coach when she was a teenager. She states that she loved him and doesn't want him to get in any trouble. Based on this disclosure, the social worker should: A. Maintain confidentiality. B. File a child abuse report because he could be doing it to others. C. Encourage client to press charges. D. Refer client to psychiatrist for medication evaluation.

A. Maintain confidentiality. If client is currently over 18 years old than you maintain confidentiality. Even if they were molested when they were younger.

In an initial session, a client reports increasing feelings of depression and hopelessness after becoming unemployed four months ago. The client exhibits increasing frustration during the session, which culminates at the end when the client yells, "What is the point of this?! I spent 2 hours with you and still don't have any money for my landlord!" The social worker clarifies the agency's services and policies, but the client is still angry. What should the social worker do FIRST: A. Refer the client to an emergency assistance program. B. Explore the client's feelings of anger. C. Assist the client in looking for employment. D. Refer the client to a medical doctor for an evaluation.

A. Refer the client to an emergency assistance program. RATIONALE: The client tells the therapist what he needs - money for rent. So the best place to start would be emergency assistance. It appears to be normal frustration, so B is not clearly indicated at this point. There is no reason for D and while C may be helpful, A is more important and needs to be addressed FIRST.

In establishing a therapeutic alliance with a client from a different racial or ethnic background, the FIRST issue for the social worker to address is: A. The racial or ethnic difference and how it may impact treatment. B. Transference issues with the therapist. C. The therapist's countertransference. D. The client's history of encounters with people of different racial or ethnic backgrounds.

A. The racial or ethnic difference and how it may impact treatment. RATIONALE: The best answer option is A because it deals directly with the client-therapist relationship and brings the topic up for exploration. B might be done later in treatment, but not FIRST. C would be addressed in consultation or supervision if the therapist felt it was interfering with treatment, but would not be done FIRST. D would not do anything to help establish a therapeutic alliance between the client and social worker.

A client seeks therapy with a social worker and discloses that she engages in non-suicidal cutting and has a history of sexual abuse as a child. The social worker has no experience in this area and feels uncomfortable working with the client. Social worker values require that the social worker: A. find a therapist skilled in the area to whom the client can be referred. B. read literature on multiple personalities to better help the client. C. help the client decrease the urges to cut. D. find an experienced social worker who would be willing to provide occasional supervision.

A. find a therapist skilled in the area to whom the client can be referred. (You haven't started a relationship yet, so you want to refer)

8. In working with a client, you become aware that she persistently behaves in ways to please or gain the approval of others. While this is not always problematic, you discover that she is obsessed with wearing the "right" clothes, living in the "right" neighborhood, and marrying the "right" person. At present, her finances are in a shambles as she tries desperately to "keep up with the Joneses," and her romantic life is suffering, as she only pursues relations that she believes others think are optimum, rather than judging relationships on more personally relevant values, such as her feelings for them, baseline compatibility, etc. Utilizing Kohlberg's Theory of Moral Development, specify the Level and Stage of moral development that applies to this individual: a. Conventional Level, Stage 3. b. Pre-conventional Level, Stage 1. c. Post-conventional Level, Stage 6. d. Conventional Level, Stage 4.

A: Conventional Level, Stage 3. The Theory of Moral Development was created by Lawrence Kohlberg, to extend and enhance Jean Piaget's theory. Overall, Kohlberg felt that the process of moral development was more complex and extended than that put forth by Piaget.

12. You are seeing a 16-year-old youth who has, for the past year, been losing his temper frequently, is regularly argumentative with adults, often refuses to follow direct requests, is easily annoyed, and routinely uses blaming to escape responsibility. Approximately four months ago he was caught in a single episode of shoplifting. The most appropriate diagnosis for this youth is: a. Oppositional defiant disorder. b. Conduct disorder. c. Impulse-control disorder. d. Disruptive behavior disorder, not otherwise specified.

A: Oppositional Defiant Disorder. Disruptive Behavior Disorder, not otherwise specified (NOS) is an umbrella term for behavior disturbances that lack sufficient clarity for the assignment of a more specific diagnosis. Impulse-Control Disorder is only appropriate when a behavior is compulsive in nature. While anger may be a part of that picture, it tends to be an overreaction to a provocation; other relevant compulsions include gambling, skin-picking, kleptomania, etc. The hallmark of Conduct Disorder is deliberate cruelty, and wanton disregard for others rights and property. This client lacks any pervasive and long-standing evidence in this regard.

13. Encopresis is defined as: a. The voluntary or involuntary passage of stool in an inappropriate place by a child over the age of four. b. The voluntary or involuntary passage of stool in an inappropriate place by a competent adult. c. Deliberate fecal incontinence only in a child over age four. d. Involuntary fecal incontinence only in a developmentally delayed adult.

A: The voluntary or involuntary passage of stool in an inappropriate place by a child over the age of four (i.e., past toilet training). This is a frequently misused term. It is most frequently applied to children and developmentally delayed adults. Adults with psychosis may warrant use of the term, although the term "fecal incontinence" is more commonly used for adults. A British literature review found only one use of the term in an adult that was not either psychotic or mentally retarded-a 1932 case of a 36-year-old diagnosed with "infantile neurosis." The most typical etiology is stool impaction (constipation) compromising sphincter control and allowing leakage into the underclothing. However, emotional disorders, anxiety, or oppositional defiant disorder can sometimes underlie the behavior. Incidence of the condition drops steadily after age six.

20. "Single system" research designs involve observing one client or system only (n=1) before, during, and after an intervention. Because of their flexibility and capacity to measure change over time, single system designs are frequently used by practitioners to evaluate: a. Their practice. b. Difficult clients. c. Conformation to policy. d. Regulation adherence.

A: Their practice. The evaluation process involves: 1) problem identification (called the "target" of the research); 2) operationalization (selecting indices that represent the problem that can be measured; 3) determining the "phase" (the time over which measurement will occur), including a "baseline phase" (without intervention) and an "intervention phase." This may also include a "time series design," where data is collected at discrete intervals over the course of the study.

A client has been drinking frequently since the death of his son last year. He states that he feels better physically when he drinks more, especially in the mid-morning, and that he does not feel intoxicated as often as he did immediately after his son's death. He expresses his desire to quit but says that he cannot get away from the craving. The client feels embarrassed because his sister found him unconscious one night as a result of his drinking. Which of the following would most likely be the social worker's FIRST diagnosis? Depression Alcohol Dependence Alcohol Abuse Avoidant Personality Disorder

Alcohol Dependence. Depression (answer #1) is not correct because while the client may be depressed, more data would be needed to make this diagnosis. Alcohol Abuse (answer #3) is not correct because the client would need to show at least two physiological factors and five behavioral patterns over a twelve-month period, which is severe and exceeds the requirements for Alcohol Abuse. Avoidant Personality Disorder (answer #4) is not correct because this question does not provide sufficient information to determine this diagnosis.

A 40-year-old male client explains that he is seeking care because he suffers from panic attacks. He notes that he gets chills, nausea, dizziness, and experiences fear about once a day. A social worker's FIRST step with this client would be to Teach relaxation therapy Set goals for reduction of anxiety Arrange for a doctor's appointment Assess the client's current situation

Assess the client's current situation

1. In determining the appropriateness of a client for a therapy group the most important single factor is A) Major events in the life history of the client. B) Social relationships with the family. C) Physical or intellectual strengths. D) Attitudes toward him/herself, adaptive operations, perception of reality.

Attitudes toward him/herself, adaptive operations, perception of reality.

Fourth stage of community building where members have a true commitment and community and have empathy and understanding

Authentic

7. The initial symptoms of schizophrenia MOST often occur between which of the following years of age? a. 6 to 12 b. 13 to 16 c. 17 to 45 d. 16 to 60

c. 17 to 45

1. According to Beck, a depressed person does not see the positive aspects of his/her situation because of: a. Precipitating factors b. Lack of support c. A dysfunctional schema d. Lack of adaptive coping strategies

c. A dysfunctional schema

All of the following are associated with a diagnosis of Hypochondriasis EXCEPT: A. "Doctor-shopping" behavior, in which the client goes from provider to provider, believing he is not getting proper medical care B. A preoccupation with physical symptoms that is of delusional intensity C. Failure to have actual medical conditions diagnosed D. A willingness to consider the possibility that no serious disease actually exists

B. A preoccupation with physical symptoms that is of delusional intensity

A 68-year-old man is referred to a social worker by his company's Employee Assistance Program for a decline in job performance. During the intake, the man expresses frustration about increased caregiving responsibilities at home after his wife's recent stroke; he reports that he hasn't been sleeping well, feels irritable toward his wife, and sometimes wishes there were someone else to take care of her. What should the social worker do NEXT? A. Assess for signs of elder abuse. B. Acknowledge the man's feelings of distress about the changes in his marital relationship. C. Refer the man for in-home caregiving services. D. Teach relaxation techniques to help combat stress.

B. Acknowledge the man's feelings of distress about the changes in his marital relationship.

During a session with a long-term client, a social worker learns that the client has been having strong suicidal ideation for the last week. The client has a tentative plan and access to means. The client is adamant about not going to the hospital. What should the social worker do NEXT? A. Initiate an involuntary hospitalization. B. Ask the client for a release to contact family members to set up a 24-hour watch. C. Provide the client with the number for a suicide prevention hotline. D. Explore the events leading up to the client's suicidal feelings.

B. Ask the client for a release to contact family members to set up a 24-hour watch.

A young adult client has been meeting with a social worker due to a diagnosis of major depression. When discussing his childhood, the client reports that he was often bullied and humiliated by peers at school. The client reports this with a flat affect and a sense of detachment. The social worker should respond by: A. Sharing a personal experience of being teased as a child. B. Assist the client in developing awareness around his feeling related to being bullied. C. Confront the client's denial. D. Provide the client with psychoeducation on the relationship between bullying and depression.

B. Assist the client in developing awareness around his feeling related to being bullied. Rationale: B is the best answer because awareness is the first step in helping the client integrate and process his feelings. A is inappropriate; C is inappropriate and may cause the client harm. D is less therapeutic than B and may not be helpful for the client.

The parents of a 7-year-old child with developmental disabilities and cerebral palsy meet with a social worker. The parents report that their child's physical and emotional needs have had a large impact on their other children and the family's day-to-day functioning. The family is well-connected to service providers, but report that they still have unmet needs. What should the social worker do FIRST? Response: A. Suggest a family meeting to discuss the needs. B. Assist the parents in prioritizing the family's needs. C. Refer the family for intensive case management. D. Schedule a meeting with all service providers.

B. Assist the parents in prioritizing the family's needs. Rationale: B is the best answer in this case because it is important to determine the issues and needs that are most pressing for the family before proceeding. A, C, and D might all be appropriate actions, but not before the social worker understands which needs are most important for the family.

A therapist at a community mental health clinic conducts an intake interview with a 20-year-old male who has been referred by his mother. He states that his mother is concerned about his lack of relationships and tendency to isolate himself in his room playing computer games. When prompted, he tells the therapist that he is lonely and wishes he could go out and meet people, but they would just be disappointed when they got to know him; he reports feeling comfortable interacting online because he knows the people "on there" will accept him. The therapist is MOST likely to treat him for: A. Major Depressive Disorder B. Avoidant Personality Disorder C. Schizoid Personality Disorder D. Panic Disorder With Agoraphobia

B. Avoidant Personality Disorder

A woman brings her 30-year-old daughter to meet with her social worker. The daughter is diagnosed with depression and lives with her mother. The mother reports that the other night, the daughter made a suicidal threat and then locked herself in the bathroom for 2 hours; the mother is now afraid that the daughter is ready to make a suicide attempt. The daughter has two previous suicide attempts and has stopped taking her antidepressants. In the interview, the social worker finds that the client is withdrawn, but calm, and unwilling to contract for safety. What should the social worker do FIRST? Response: A. Help the mother contact the daughter's psychiatrist. B. Begin to take steps towards an emergency psychiatric commitment. C. Attempt to convince the daughter to take her medication. D. Explore the daughter's recent increase in suicidal thoughts.

B. Begin to take steps towards an emergency psychiatric commitment. RATIONALE: Because the client is unwilling to contract for safety, the therapist must consider B. Her safety is the most important issue and her unwillingness to sign contract is an indication of high risk for suicide.

A 25-year-old shows up for a therapy session high. The client is still coherent, and able to speak with the social worker and answer questions. What should the social worker do FIRST? A. Refuse to see the client. B. Confront the client's inappropriate behavior. C. Encourage the client to attend Narcotics Anonymous. D. Initiate a behavioral intervention to address this behavior.

B. Confront the client's inappropriate behavior. Rationale: B is the best answer because until the behavior is confronted, other interventions will likely be ineffective. A, C, and D might all be useful interventions, but not BEFORE the behavior is addressed directly.

A social worker receives a subpoena from the court requesting a current client's record. The social worker believes that releasing the file would cause great harm to the client. After contacting the client, the social worker should NEXT: A. Release the records to the court. B. Contact the court and advocate for confidentiality of record. C. Exert privilege on behalf of the client. D. Release only the part of the record relevant to the case.

B. Contact the court and advocate for confidentiality of record.

A school social worker meets with a sixth grade teacher regarding a student who was recently diagnosed with Type I diabetes. The teacher reports that the student has isolated herself from the other students, has fallen behind, and isn't interested in participating in class like she used to be. The social worker FIRST: Response: A. Meet with the student and explore her feelings about receiving the diagnosis. B. Contact the student's parents to obtain additional information. C. Help the teacher understand the effects of being diagnosed with a long-term illness during this critical stage of identity development. D. Refer the student to a support group.

B. Contact the student's parents to obtain additional information. Rationale: B is the best answer in this case because the social worker needs to understand more about the student's overall functioning and adjustment to the diagnosis; A and D might be useful, but not until after the social worker contacts the parents for more information. C will likely not affect the student's behaviors.

A social worker has been meeting with a 9-year-old child who was severely neglected by her parents. The child is in foster care and has been seeing the social worker for six months with good results. The social worker discovers that the child's parents want to get her back, however, refuse to follow the treatment plan mandated by the judge. They report that they know what they did was wrong, but state that since they are her parents they deserve to have her back. The social worker should focus on: A. A plan for reunification. B. Developing a permanency plan for the child. C. The parents' mental health treatment. D. Helping the parents change their behavior.

B. Developing a permanency plan for the child. Rationale: B is the best answer because the child's parents have had sufficient time to make changes and address their issues, but have failed to do so; at this point, the social worker should focus on finding a permanent, safe place for the child. A is not a good choice, since the parents have not met their court-ordered obligations to achieve this. C and D don't make sense in this context, since the social worker's responsibility is to the child.

A client with a history of anger issues and aggressive behavior has been seeing a social worker for therapy for two months. During a heated session, the client storms out. The following day he delivers a letter demanding his records or he will take legal action. The social worker believes that releasing his records will cause him serious harm. According to professional ethics, the social worker should A. Seek legal consultation regarding the threat to sue. B. Document in the record both the request and the rationale for non-release. C. Release the full record as requested. D. Schedule a session to discuss the client's desire to obtain records.

B. Document in the record both the request and the rationale for non-release.

Piaget believed that children in the preoperational stage have difficulty taking the perspective of another person. This is known as: Response: A. Reversibility B. Egocentrism C. Metacognition D. Constructivism

B. Egocentrism RATIONALE: B is the best answer because egocentrism refers to the child's focus on their own needs and perspective to the exclusion of others.

A social worker in private practice meets with the parents of a young child who has recently been diagnosed with Asperger's Disorder. The parents report that the child's teacher is open to working with the child and they feel supported by the school and community. The social worker should NEXT: Response: A. Recommend that the parents advocate for a teacher's aide to assist with the child in class. B. Encourage the parents to enroll the child in a social skills group. C. Refer the parents to a gymnastics class for children with special needs. D. Help the parents identify appropriate classmates for future play dates.

B. Encourage the parents to enroll the child in a social skills group. Rationale: B is the best answer because it will help the student prepare for both C and D. A is not supported by the information in the stem; the student and teacher may not need a teacher's aide.

A 45-year-old man meets with a social worker for an initial appointment. He reports that his wife is an alcoholic and becomes verbally abusive when she is intoxicated. He indicates that his wife refuses to go to treatment, insisting that they can resolve this problem on their own. The therapist should FIRST: A. Provide the man with psychoeducation around substance abuse and the stages of change. B. Explore the man's issues and work collaboratively to develop treatment goals. C. Explore the man's commitment to the relationship. D. Help the man identify the reasons behind his wife's behavior.

B. Explore the man's issues and work collaboratively to develop treatment goals. RATIONALE: The best answer is B. A, C, and D could be options later in treatment after a treatment plan focusing on the man has been established.

A 27-year-old male seeks treatment from a social worker after his parents demanded that he move out of their house. The client is confused because his parents would still like him to come over for dinner several times a week and have encouraged him to bring his laundry home for his mother to do. When the client suggests including his parents in a session, they decline, stating that he is an adult and does not need them. The social worker should FIRST: A. Encourage the client to separate himself from his parents. B. Explore the parent-child relationship and the events leading up to his parents asking him to move out. C. Address dependency issues. D. Explore the client's difficulty in moving out of his parents home.

B. Explore the parent-child relationship and the events leading up to his parents asking him to move out. RATIONALE: B is the best place to start. Exploring the issue further will give you important information about what to focus on. There is not enough information to address C or D and the client is not indicating an interest in A at this point.

A social worker is meeting with the parents of a 7-year-old boy who was recently diagnosed with ADHD. The parents met with a psychiatrist who recommended medication and behavioral management therapy. The parents are resistant to medication. What should the therapist do first? A. Refer them back to the psychiatrist. B. Explore their concerns about medication. C. Begin behavioral modification training. D. Encourage them to try out medication before they judge it.

B. Explore their concerns about medication. RATIONALE: B is the best answer because it's important for the social worker to explore the parents' concerns about medication and what is behind that concern before further steps are taken.

A 17-year-old client reports that her boyfriend recently hit her in the face during an argument, but since then he has been really nice and bought her an expensive watch. She says her parents told her to break up with him, so she lied and told them she did. What should the social worker do NEXT? A. Encourage the client to break up with the boyfriend. B. Explore what it is like to have to lie to her parents. C. Encourage her to be honest with her parents. D. Inform the client that the boyfriend's actions are an example of the honeymoon phase.

B. Explore what it is like to have to lie to her parents.

A mother and her 8-year-old daughter are referred to a social worker after a school counselor receives several reports that the girl has been bullying classmates. The mother is confused and says that her daughter gets along well with her two siblings and other children in their apartment complex. After completing a social and developmental history from the mother, the social worker should NEXT: A. Schedule an individual session with the girl. B. Have the mother sign a release to talk with the school counselor. C. Schedule a session with the siblings to gather additional information. D. Request the girl's school record.

B. Have the mother sign a release to talk with the school counselor. Rationale: B is the best answer because this appears to be a school-based issue and the collateral information from the school is necessary for assessment. A and C might both be good steps after the collateral information from the school is obtained. D is not the best answer because reviewing the school records will not provide as much information as contact with the counselor.

6. How do you find the median of a set with an even number of items? a. The larger of the two middle numbers is the median. b. The two middle numbers are both medians. c. Find the average of the two middle numbers. d. A set with an even number of items does not have a median.

c. Find the average of the two middle numbers.

A client with a diagnosis of major depression meets with a social worker. The client reports that she has been in treatment many times before, but says, "my depression has never gotten any better." While developing the treatment plan, the client agrees with the social worker's suggested objectives saying, "I will do anything to make this go away." To develop a realistic treatment plan, the social worker should FIRST: A. Ensure the client's understanding of the treatment process. B. Help the client identify barriers to success in previous treatment attempts. C. Suggest that the client set her own objectives. D. Discuss evidenced-based treatment protocols for major depressive disorder.

B. Help the client identify barriers to success in previous treatment attempts. Rationale: B is the best answer because it is the most effective way to improve the chances of success in this treatment relationship. A, C, and D may all be part of the treatment planning process, but would not be done before the therapist understands the barriers to previous attempts at treatment.

A 38-year-old woman meets with a social worker after having recently found out that her husband has a rare type of cancer. During the initial meeting, the client expresses little emotion and instead focuses on the success rates of various medical treatments for her husband. This is an example of: A. Rationalization B. Intellectualization C. Displacement D. Sublimation

B. Intellectualization RATIONALE: B is correct because intellecutalization is a defense mechanism where the client avoids uncomfortable emotions by focusing on the facts which in this case is the medical treatments and success rates.

A couple and their two children, ages two and four, come in for family counseling. They tell the social worker that the kids are "out of control". The father has a long history of alcohol abuse and the mother suffers from frequent depression. The social worker should consider referrals to: A. A child protective agency, AA, and a physician. B. Parenting classes, AA, and a psychiatrist. C. Parenting classes, substance abuse treatment, and a psychiatrist D. Substance abuse treatment, a physician, and a psychiatrist.

B. Parenting classes, AA, and a psychiatrist. Rationale: B is the best answer because it addresses the needs that are presented in the stem: parenting difficulties, alcohol abuse, and the mother's depression. A is not the best answer because there is no indication of child abuse in the stem. C is a good answer, but not as strong as B, which specifies AA instead of just general substance abuse treatment. D is not the best answer because there are no issues requiring physician care presented in the stem.

A social worker receives a court-ordered subpoena for a former client's records. The social worker is concerned that the release of the records will cause harm to the client. The social worker advocates to the court to limit the scope of the release. The court declines the request. The social worker should NEXT: A. Contact the client and warn them of the pending release. B. Provide a copy of the records immediately. C. Consult with a lawyer regarding the matter. D. Limit the release to only pertinent information.

B. Provide a copy of the records immediately.

A toddler is referred to a social worker because he is difficult to soothe and aggressive towards his other siblings. The mother reports that both her and her husband are gone a lot due to work commitments, but have hired multiple caregivers to watch their toddler when they are not home. What should the social worker do FIRST? Response: A. Encourage the parents develop a more regular work schedule. B. Provide psychoeducation to parents on the importance of their involvement with the child. C. Provide referral to a preschool. D. Refer child to a psychiatrist.

B. Provide psychoeducation to parents on the importance of their involvement with the child. RATIONALE: Psychoeducation is the best option in this case as it appears that the child is responding to environmental stress. A may not be possible. C and D could be done after that.

A hospital social worker is working with a 35-year-old client who was involuntarily committed to the psychiatric unit due to the risk of self-harm. The client has a long-term history of anorexia nervosa and, as a result, is experiencing many medical complications. While meeting with the social worker, the client states that she does not want to be in the hospital, and has been refusing the hospital food. The client asks to be referred to a day treatment program. What should the social worker do FIRST? A. Refer the client for a medication evaluation. B. Remind the client that her hospitalization is not voluntary. C. Support the client in her efforts to be discharged. D. Point out the impact of the client's behaviors on her overall health.

B. Remind the client that her hospitalization is not voluntary. Rationale: B is the best answer because the client is on an involuntary hold and has few options at this time. A may be needed, but is likely already part of the treatment protocol at the hospital. C is not the best answer because the client appears to be at risk of self-harm. D might be a good option, but not before discussing the nature of the hospitalization.

During the initial session, the client appears fearful and anxious and blurts out "I need to be sure that everything I tell you will stay between us." The social worker should FIRST: A. Validate the client's fear and anxiety because therapy can be scary. B. Review with the client the limits to the social worker/client confidentiality. C. Remind the client that it is the social worker's responsibility to earn the client's trust. D. Assure the client that everything they discuss will be kept confidential.

B. Review with the client the limits to the social worker/client confidentiality.

A family meets with a social worker after their 17-year-old son was diagnosed with schizophrenia. The psychiatrist recommends intensive outpatient treatment so that the client can remain in the family home with a support system. The family physician believes that inpatient treatment is necessary due to the presence of alcohol abuse. The client's parents are afraid and unsure what to do. The social worker should FIRST: A. Support the family in making a treatment decision. B. Schedule a conference with both the psychiatrist and physician involved. C. Try outpatient therapy with the addition of AA meetings. D. Discuss the importance of sobriety with the family.

B. Schedule a conference with both the psychiatrist and physician involved. Rationale: B is the best answer in order to help the family and achieve some agreement between professionals regarding the course of treatment. A and D might both be reasonable, but not until after trying to obtain consensus between the professionals involved. C disregards the family physician's opinion, which may be based on physical withdrawal symptoms associated with alcohol abuse.

A 45-year-old married mother of two self-refers to a therapist in private practice. She reports a strong desire to return to the workforce now that her children are in high school but reports that she's been canceling her scheduled interviews due to panic and fear that she will embarrass herself during the interview. What is the MOST likely diagnosis? A. Panic Disorder B. Social Phobia C. Agoraphobia Without History of Panic Disorder D. Anxiety Disorder NOS

B. Social Phobia

A therapist begins treatment with a child diagnosed with ADHD and intends to use cognitive behavioral treatment to help improve the child's concentration. The therapist will MOST likely need to help the child in which additional area? A. Expressive language B. Social skills C. Basic hygiene D. Bladder control

B. Social skills

A hospital social worker is called to the emergency room to evaluate a homeless man who was brought in by police. The social worker finds the man in restraints and is told that the patient has been threatening to cut himself intermittently since he was admitted. The patient tells the social worker that he needs to use the bathroom. What should the social worker do? Response: A. Immediately evaluate the patient for risk to self. B. Speak with the charge nurse regarding the patient's request. C. Have a staff member accompany the patient to the restroom. D. Inform the patient that he will have to contract for safety before going to the bathroom.

B. Speak with the charge nurse regarding the patient's request. RATIONALE: The social worker is part of the hospital team and should coordinate with their plans. B is the best place to start to ensure that the client is not attempting to split the staff.

The parents of a 6-year-old boy who was diagnosed with autism meet with a social worker. During the session, the parents complain that the school does not provide a classroom that caters to their child's needs. The social worker should: A. Advocate for the child by speaking with the school administration. B. Support the parents in scheduling a meeting with the school administration. C. Schedule a meeting with the child's teacher to discuss a behavioral plan. D. Provide the parents with behavioral suggestions that can be implemented at home and shared with the teacher.

B. Support the parents in scheduling a meeting with the school administration. RATIONALE: Social worker's should empower clients to advocate for themselves so B is the best answer. A and C do the work for the clients. D could be done after B.

In an initial interview, a client appears suspicious of the agency and the social worker. He says that he wants to reveal certain facts but feels that this would get his friends in trouble. The appropriate response for the social worker is to: A. Discuss a less threatening subject with the client. B. Tell the client that she understands how hard it is for him to feel safe in this new situation and that he should reveal information when he feels he is ready. C. Explain to the client how social workers maintain confidentiality and show him how the files are locked to ensure privacy. D. Encourage the client to talk about these things since he can trust the social worker to maintain confidentiality.

B. Tell the client that she understands how hard it is for him to feel safe in this new situation and that he should reveal information when he feels he is ready.

A patient admitted into ER has his partner listed as the emergency contact. The social worker learns that the nursing staff contacted his family instead of the partner. The social worker should NEXT: A. Call the partner. B. Tell the patient that his family was notified. C. Wait until the family arrives and find out what they think is best. D. Discuss the situation with the doctor.

B. Tell the patient that his family was notified. RATIONALE: B is the best answer because the client should know and be able to emotionally prepare for his family's response or arrival at the hospital. A might be a good step, but not until after the client is informed of the mistake. C would violate the client's confidentiality and takes away his self-determination. D does not make sense since the doctor has not bearing on this particular situation.

While meeting with a social worker, a client discusses his concerns regarding the amount of time he spends playing videogames. The client reports that he has trouble paying attention to "anything else" because he's preoccupied with strategy and planning associated with the games. He states that he has tried, "but can't control it." The social worker should FIRST assess? A. How the content of the games contributes to the client's preoccupation. B. The specific ways in which the client's life is impacted. C. The quality of the client's interpersonal relationships. D. The frequency and duration of his gameplaying.

B. The specific ways in which the client's life is impacted. Rationale: B is the best answer because the social worker needs a more complete understanding of the presenting problem and its impact on the client's life. A, C, and D might all be part of the overall evaluation, but would not be clarified before the social worker understands the extent of the problem.

A client meets with a social worker and states that she has a medically frail child who is 3-years-old. The client states that she feels hopeless that things will change, and is angry at how it's affecting her family. The client feels that the needs of this child are impacting the client's other children who are oppositional and having problems in school. What should the social worker do FIRST? A. Refer the client to a support group for parents of children with medical problems. Should have chosen B. Work with the family as a unit. C. Coordinate with school social worker to address problems with other children. D. Work with the client around ways to reduce her stress.

B. Work with the family as a unit. RATIONALE: The problem is affecting the whole family, so that is the best method of treatment. A, C, and D could all be done as part of the treatment.

5. Identify the missing step in Albert R. Roberts seven-stage crisis intervention model: 1) assess lethality; 2) establish rapport; 3) __________; 4) deal with feelings; 5) explore alternatives; 6) develop an action plan; 7) follow-up. The third step is: a. Evaluate resources. b. Identify problems. c. Environmental control. d. Collateral contacts.

B: "Identify problems" is the third step in the Roberts crisis intervention model.

11. The Diagnostic and Statistical Manual of Mental Disorders (DSM), in its current version (the DSM-IV-TR), is used to diagnosis mental disorders. It also allows for the entry of related factors through a multi-axial coding approach. Name the proper axis (in the corresponding order) for entry of each of the following: a) the Global Assessment of Functioning; b) relevant physical disorders; c) personality disorders and mental retardation; d) relevant psychosocial factors; and e) clinical disorders: a. Axis I; Axis III; Axis II; Axis IV; and Axis V. b. Axis V; Axis III; Axis II; Axis IV; and Axis I. c. Axis II; Axis IV; Axis III; Axis V; and Axis I. d. Axis IV; Axis III; Axis II; Axis V; and Axis I.

B: Axis V; Axis III; Axis II; Axis IV; and Axis I. Clinicians should assess clients on all five axes to ensure a thorough evaluation.

16. You are called to see a young black man in his mid-twenties. Two adult sisters brought him for an urgent appointment. The young man is clean, neatly dressed in slacks, dress shoes, and a tweed sport coat. He is also calm, relaxed, and without any signs of agitation. The two sisters, however, appear disheveled, frazzled, and almost histrionic. They blurt out the he "has problems" and urge you to talk with him. Privately, he tells you that he is fine. Later, however, the ladies tell you he left home abruptly and traveled cross-country with no destination. He didn't sleep for three days (with them pursuing him), was spending money excessively and writing checks he couldn't cover. He ended up in a nationally famous amusement park at 3:00 a.m. (having scaled a fence), sitting on an empty roller coaster "waiting for the ride to start." When confronted, he admits all of this, but says he's now rested, and doing better. The most likely diagnosis would be: a. Brief psychotic disorder. b. Bipolar I, single manic episode, in full remission. c. Bipolar I, single hypomanic episode, in full remission. d. Cyclothymic disorder.

B: Bipolar I, single manic episode, in full remission. There is no evidence of frank psychosis, thus brief psychotic disorder can be ruled out. Hypomania does not appear appropriate, as the client's behavior would likely have resulted in hospitalization had anyone been able to evaluate him during his period of mania. Cyclothymic disorder does not appear appropriate, as the client's conduct exceeded the threshold severity for hypomania, and no information is provided regarding depressive symptoms (though he may well have them). Finally, the Bipolar I, single manic episode is identified to be in full remission, as the client's manic symptoms appear to have completely resolved.

17. A therapeutic approach that views the client from a social context, that sees behavior as derived from unconscious drives and motivations, that views disorders and dysfunction as emerging from internal conflicts and anxiety, and that seeks to facilitate the conscious awareness of previously repressed information is called a: a. Cognitive approach. b. Psychoanalytic approach. c. Gestalt approach. d. Behavior approach.

B: Psychoanalytic approach. This approach is built upon the concepts and theory of Sigmund Freud and others who have followed him. The approach is also sometimes called a "psychodynamic" approach.

3. You are hired by a private practice therapist who operates a court-supervised violent offender treatment program. One of your responsibilities is to screen new client referrals, to ensure that only low-risk, first-time offenders are accepted into the program. In this process, you are to have each client sign a treatment consent form, which also includes a detailed consent for release of information. You note that instead of the usual time and target limits, the form allows information to be released at any time to "any law enforcement agency," "any spouse, ex-spouse, or significant other," "any welfare or abuse protection agency," etc. You ask about the ethics of having clients sign this form, and you are told, "It's a hassle to try and get specific information releases, and the safety of the public is at stake. Use the form." Your BEST response is to: a. Use the form as directed. b. Refuse to use the form. c. Call your licensing board and discuss the form. d. Call law enforcement and discuss the form.

B: Refuse to use the form. No client or client population is beneath the ethical standards of the field. An appropriate information release form stipulates a limited period of time beyond which the form expires, the specific kind of information to be released, the specific purpose for which the information is to be provided, and a specific individual or entity to whom/which the information will be provided. While obtaining an information release is indeed a "hassle" it is the ethical standard of care in the field, and deviation from it can open a practitioner to legal liability. The fact that a given client, or client population, may be unaware of this does not excuse the therapist from using an ethically appropriate form in keeping with expected standards of care. Any limitations to confidentiality-such as mandatory reporting if a client expresses intent to commit a crime or harm another-belong on a treatment consent form, rather than on an information release form.

What should an infant be doing at 5 months?

Begin to enjoy vocal play, grasp objects, put objects in mouth

behaviors learned through conditioning and we must help teach new skills and behaviors

Behavior theory

Type of addiction model which says all other models are true to greater or lesser degrees. Each person's drug use is result of some aspects of some or all other models. Treatment and recovery require addressing the body/mind/social environment and spiritual needs of an individual.

Bio-psycho-social model

Physical development: - age 1-4 months head circumference should grow at a rate of about 1 and half inches for the first two months of life, and then at about an inch per month until an age of 4 months. Gums red.

Brain is growing - great for development

A social worker on an Assertive Community Treatment (ACT) team is working with a 19-year-old male who is preparing for discharge from an inpatient unit. In order to return to his independent living situation, the client agrees to work with the social worker to create a behavioral contract to prevent rehospitalization. The MOST important item in the contract is: A. A list of adaptive coping skills to utilize during times of stress. B. An agreement to maintain control. C. A plan to seek help before loss of control. D. Potential consequences of contract violation.

C. A plan to seek help before loss of control. RATIONALE: Since the point of the contract is to prevent future crises, the most important pieces of the contract are the steps the client will take to prevent a loss of control. A and D will be part of the contract, but are not more important that the prevention plan; B is basically meaningless without a specific plan for how the client will respond to stressors.

A social worker meets with a client who has a history of DUI and reckless endangerment of his two small children. The client has followed his court-ordered service plan including regular attendance at individual and group treatment sessions, as well as regular drug testing and mandatory employment. In an individual session, he tells his social worker that he wants to see his two children, who are currently in custody with his ex-wife. The social worker should NEXT: A. Speak with the ex-wife regarding the father's wishes. B. Ask the client if the two children want to spend more time with him. C. Address any legal barriers to visitation. D. Schedule a supervised visit for the father.

C. Address any legal barriers to visitation. RATIONALE: The first thing would be to see if there is any reason he can't see his kids C. After that was determined, A, B, or D could be considered.

A new client calls seeking therapy for depression. On the phone, she reports feelings of hopelessness and doubts about therapy. How should social worker proceed? A. Validate her feelings. B. Schedule an appointment. C. Ask if client has any thoughts of harming herself. D. Explore her doubts about therapy.

C. Ask if client has any thoughts of harming herself. Hopelessness is a sign of suicide. Need to find out.

A school counselor refers a family to a social worker after finding their 7-year- old child pretending to have sexual intercourse with another child at recess. The mother states, "I'm so embarrassed about this happening at school, and not sure what to do." What should the social worker do FIRST: A. Report suspected child abuse to the proper authority. B. Provide psychoeducation on normal child development. C. Assess for further signs for sexual abuse. D. Consult with the school counselor to obtain further details about the incident.

C. Assess for further signs for sexual abuse. RATIONALE: This is outside of normal behavior for a 7-year-old, so B is out. Ruling our sexual abuse would be the first step. A is premature and D could be done after talking with the parents.

A client with a history of major depression who has been in therapy for ten months comes to session and abruptly informs the social worker that he feels much better and will be ending therapy that day. He thanks the social worker for all the help and gives the social worker a favorite painting. What should the social worker do NEXT? A. Refuse the painting because it is a conflict of interest. B. Explore the significance of the painting. C. Assess if the client has any thoughts of hurting himself. D. Share concerns about the abruptness of termination.

C. Assess if the client has any thoughts of hurting himself.

A 30-year-old single female is in treatment with a social worker. She hates her job working for her family but consistently demonstrates an inability to take any initiative in her life because she's sure she'll fail. She doesn't want anyone to know she's in therapy because she's afraid of what they'll think of her. According to Erik Erikson, this is an example of which unresolved developmental stage? A. Trust vs. mistrust B. Identity vs. role confusion C. Autonomy vs. shame and doubt D. Initiative vs. guilt

C. Autonomy vs. shame and doubt RATIONALE: C is the best answer because the stem describes a client who is having a hard time separating from her family and has feelings of doubt and shame. A, B and D would all be described differently.

A social worker meets with a lesbian couple seeking help with the decision of getting married and starting a family. After two sessions, the social worker asks questions regarding their sexual relationship. The clients appear uncomfortable and ask why the social worker wants to know. The social worker should first explain that: A. It is common for couples to experience sexual problems. B. Exploring their sexual history is an important aspect of addressing underlying issues. C. Beliefs surrounding sex are an important part of the relationship to address. D. It is important to address any sexual problems now before they escalate.

C. Beliefs surrounding sex are an important part of the relationship to address. RATIONALE: C is the best answer and is related to why they are coming for therapy. A, B, and D suggest there is a problem.

A social work supervisor receives a subpoena from the court to testify in a court hearing for a client accused of child abuse. The social worker should FIRST: A. Assert privilege. B. Contact the client to determine the client's wishes. C. Consult with a lawyer regarding the matter. D. Review the client's records to determine what to say.

C. Consult with a lawyer regarding the matter.

A social worker is in the process of creating a multidisciplinary team for a new mental health clinic. In order to facilitate collaboration between team members, the social worker should: A. Reinforce the importance of including social work as a component. B. Identify the population of clients that the team will serve. C. Define each member's contribution to the overall team. D. Identify one member to act as the team leader.

C. Define each member's contribution to the overall team. Rationale: C is the best answer because it will lay the foundation for any step taken afterward in terms of teambuilding and ensure that team members can work together; A, B, and D do not address the task of creating a collaborative team.

Schizoaffective Disorder and Mood Disorder With Psychotic Features share all of the following features EXCEPT: A. Psychotic symptoms occurring concurrently with mood symptoms B. Significant mood disturbance C. Delusions and hallucinations in the absence of prominent mood symptoms D. The disturbance is not due to the direct effects of a medical disorder or substance

C. Delusions and hallucinations in the absence of prominent mood symptoms (because schizoaffective needs a mood disorder)

A 14-year-old has been referred to the school social worker because of hostility and fighting with peers. To establish a therapeutic relationship with this client, the social worker should FIRST: A. Validate the student's hostile feelings and explore reasons for fighting. B. Advocate with the school administrators to change the student's school program. C. Discuss confidentiality limitations with the student. D. Reduce the student's tension by discussing school activities.

C. Discuss confidentiality limitations with the student. -Important to talk about confidentiality before going into therapy.

A crisis social worker receives a call that a man is walking through a local park naked. It has also been reported that he was seen shouting incoherently outside his home, and appeared disheveled and disoriented. His neighbors are concerned because he is a hard-working, successful individual. What should the social worker do NEXT? A. Start making phone calls to inpatient treatment programs. B. Make a home visit and complete a complete evaluation. C. Dispatch a crisis team to perform a mental status exam and arrange appropriate follow up. D. Encourage the caller to make an APS report and call police.

C. Dispatch a crisis team to perform a mental status exam and arrange appropriate follow up. Rationale: C is the best answer because the man appears to be exhibiting an inability to care for himself and may be at risk of harming herself. A does not make sense before an assessment has been made. B is not the best answer because it is likely not the crisis social worker's role to make direct visits, but rather to arrange assessment when needed. D is not the best answer because it may delay assessment and treatment, creating a safety issue.

A social worker receives a letter from a personal injury lawyer requesting information about one of the social worker's clients' diagnosis and treatment. The letter indicates that the client is the defendant in an upcoming court case. What should the social worker do FIRST? A. Contact the lawyer and explain that a subpoena will be needed to release information. B. Contact the lawyer and release selected information from the client's record. C. Do not respond to the letter and contact the client to inform them of the request. D. Send a letter to the lawyer refusing the request.

C. Do not respond to the letter and contact the client to inform them of the request. RATIONALE: A, B, and D all acknowledge that the client is you client, which is a violation of confidentiality. C is the only option.

A social worker begins the intake process with a new client, a 33-year-old female. Before the social worker can finish explaining the risks and benefits of treatment, the client jumps in and says, "Oh, I just know this is going to work! You're so much better than all of my previous therapists." After acknowledging the client's positive reaction, what should the social worker do next in order to engage the client? A. Begin negotiating the agency's treatment contract. B. Ask what interfered with previous treatment. C. Explore expectations regarding this helping relationship. D. Elicit information regarding the client's immediate attraction to the social worker.

C. Explore expectations regarding this helping relationship. RATIONALE: C is the best answer because it starts where the client is and addresses her expectations of therapy. D and B are not as directly related to engaging the client in the treatment process. A can be done after C.

A family seeks therapy for help with their teenage children. The parents recently got married and moved in together. The husband's children are ages 7 and 13. The wife's children are 9 and 14. The parents report that the teenagers are not listening and acting out. The social worker's initial intervention should be: A. Determine when the problematic behavior started. B. Obtain a release to speak to the school about their behavior. C. Explore how each of the family members are adjusting to the new living arrangements. D. Provide psychoeducation about normal adolescent development and behavior.

C. Explore how each of the family members are adjusting to the new living arrangements. RATIONALE: The INITIAL intervention should be for the social worker to explore the effects of the new living arrangements and how each family member is adjusting (C). A, B, and D may be important only after C has been done.

A social worker is using a crisis intervention model to help a family whose son was recently expelled from high school. The family is overwhelmed, does not know how to proceed, and is having significant difficulties around communication. After completing an assessment and establishing rapport with the family, the social worker should NEXT: A. Teach the family effective communication techniques. B. Refer the family to a support group. C. Explore the "last straw" before the crisis occurred. D. Set up a meeting with school administration to discuss reinstating the son.

C. Explore the "last straw" before the crisis occurred. Where to go with Crisis: Establish repport with family Find out what brought on the crisis

A social worker at a day treatment facility for teens with mood disorders is asked to provide crisis intervention following the suicide of a program participant. The social worker should A. Refrain from sharing information due to confidentiality. B. Contact the residents' parents to inform them of the situation. C. Facilitate a group session to discuss the incident with the other participants. D. Have individual meetings with each participant to discuss the incident.

C. Facilitate a group session to discuss the incident with the other participants. RATIONALE: Debriefing by giving everyone the same information is the best place to start so C is the best answer. A is something to factor in, but would be done throughout. B and D could be done after C.

An established client discloses to her social worker that several years ago, she had an affair and subsequently had a child. The client confessed to her husband and the two have been raising the child as their own since then. The client says that she never told anyone else and that she and her husband have not discussed the paternity of the child since her confession. Which family therapy concept does this illustrate? A. Family rules B. Enmeshed boundaries C. Family secrets D. Emotional cutoff

C. Family secrets Rationale: This is an application question; C is the best answer because the scenario fits the definition of "family secrets." A, B, and D would all be described in a different way.

A couple meets with a social worker. The woman reports that her 80-year-old father has been living with them since her mother passed away four years ago. The woman reports that her father has recently been refusing to eat meals with the couple, appears agitated, and is concerned that he may be depressed. The woman says that she and her husband are constantly arguing about whether or not they should suggest that her father move to an assisted living facility. The FIRST factor the social worker should consider is the: A. Family's cultural background and the value placed on caregiving. B. Father's decreased risk for suicide since he is living with a family member. C. Guilt associated with caregivers suggesting that the family member move out. D. Daughter's ability to withstand the father's negative response.

C. Guilt associated with caregivers suggesting that the family member move out. Rationale: C is the best answer because the guilt may both prevent the couple from reaching a consensus and be adding to the tension between them; without resolving it, it will be difficult to move forward. A may be considered, but not before the more likely issue of guilt. B is not the best answer because the stem doesn't provide enough information to determine whether there is a risk of suicide. D is not the best answer because the stem does not indicate that the father will have a negative response.

A treatment team meets to review patient progress on an inpatient unit. One particular patient with refractory Major Depressive Disorder is refusing to try electro-convulsive therapy, but is continuing to take antidepressants. The patient cannot be discharged due to the severity of symptoms. What is the BEST approach for the social worker to recommend to the treatment team? A. Obtain a court order to ensure that the patient will take the new medication. B. Develop a behavior modification plan requiring more participation in social and physical activities on the unit. C. Have the treatment team and patient meet to review the treatment plan. D. Obtain consultation from a psychiatrist outside of the hospital.

C. Have the treatment team and patient meet to review the treatment plan. RATIONALE: The patient should be included in the discussion of the treatment plan because his actions are impacting the course of treatment. There is no reason to get a court order to take new medication and B and D are unrelated to the issue.

A social worker receives a referral for a 13-year-old who was sexually abused by his older cousin. Collateral contact with the child protective services worker reveals that both the client and cousin are still living in the same home, although they no longer share a bedroom. After completing the evaluation, the social worker should focus on: A. Requiring the cousin to attend individual treatment. B. Working with the parents to find alternative living arrangements for the cousin. C. Helping the client develop a safety plan. D. Facilitating a family session with the client, parents and cousin.

C. Helping the client develop a safety plan. Rationale: C is the best answer because safety is the primary objective for the child. A is not the best answer because the offender is not the social worker's client. B may not be what the family wants or is able to do. D would be inappropriate and is not supported by research (treating a victim and perpetrator together).

All of the following are indicators of possible child sexual abuse EXCEPT: A. Excessive masturbation that has a driven quality B. Night terrors C. Infrequent attendance at school D. Psychosomatic illness

C. Infrequent attendance at school

All of the following are parental signs of possible child abuse except: A. Unrealistic expectations of child B. Indifference to child C. Marital discord D. Low frustration tolerance

C. Marital discord

A school social worker is working with a 14-year-old student. During one session, the student admits to having fantasies about shooting several classmates who have bullied him in the past. When the social worker clarifies his feelings, he says, "I would never act on it. I just, you know, think about it sometimes." What should the social worker do NEXT? A. Notify the student regarding the threat. B. Explain that the client's parents have to be contacted to discuss his statements. C. Monitor the client's feelings during future sessions. D. Refer the client for a psychiatric evaluation.

C. Monitor the client's feelings during future sessions.

A 7-year-old child who was sexually abused by a family friend is being treated by a social worker. During one session, the client's mother reports that his teacher contacted her to report that the child was sexually act out with another child at school. The mother reports that she is concerned by this behavior and unsure what to do. The social worker should explain that this behavior: A. Will almost certainly decrease over time. B. Probably means the child is still being abused. C. Necessitates continued treatment. D. Substantiates child abuse.

C. Necessitates continued treatment. Rationale: C is the best answer because the behavior indicates that the child has not resolved sexual issues and needs continued intervention. A and B may not be at all true; D does not make sense since the validity of the abuse was not in question.

A 34-year-old single female is referred to a therapist by her real estate company's Employee Assistance Program. She has missed a significant amount of work since her mother's death 6 months ago and has been unable to complete previously easy tasks. During the assessment, the woman reports that she spends hours every day preparing food and repeatedly cleaning her house. She adds that she can't stop thinking that the house isn't clean enough to meet her mother's standards. What diagnosis should be considered? A. Generalized Anxiety Disorder B. Bereavement C. Obsessive-Compulsive Disorder D. Obsessive-Compulsive Personality Disorder

C. Obsessive-Compulsive Disorder

During an initial evaluation, a 53-year old man reports that he is upset regarding his sex life with his wife. He reports that they used to have an active sex life, but that since his wife started menopause a few months ago, she often refuses sex or says she isn't in the mood. The client has done online research that has normalized his wife's experience, but he still believes that she may be having an affair. The social worker's MOST appropriate action is to: A. Suggest that the wife have a medical evaluation. B. Provide psychoeducation about the affects of menopause on desire. C. Offer couples therapy to the client and his wife. D. Suggest individual therapy to help the husband address his frustration.

C. Offer couples therapy to the client and his wife. Rationale: C is the best answer because it will address the issue that exists BETWEEN the husband and wife. A is not the best answer because there is not enough to suggest that this is necessary at this time. Likewise, B may not be particularly helpful in this case as the husband has done some research and is more concerned with his wife's fidelity. D is not the best answer because it would not address one partner in an issue that is between the couple.

When working with a client who is paranoid, which of the following actions could BEST be used by a social worker to convey acceptance? A. Teaching relaxation B. Maintaining constant attention and direct eye contact C. Offering assurance and gentle guidance D. Verbally reframing what the client has said

C. Offering assurance and gentle guidance

A social worker is meeting with a recently retired woman who is seeking counseling due to feelings of irritability and poor frustration tolerance. The client reports that she was looking forward to retirement so that she could do the things she enjoyed and see more of her spouse after a demanding career in the health care industry. The client says that instead she's been having trouble sleeping and just wants to be alone. She says that she doesn't enjoy the things that she used to and is confused. The social worker should FIRST inquire about: A. Any dysfunctional changes in the marital relationship. B. The impact of losing status in the community. C. Other symptoms that might indicate depression. D. Volunteer opportunities that might help the client feel useful again.

C. Other symptoms that might indicate depression. Rationale: C is the best answer because the client has indicated several symptoms of depression and the therapist should start by asking more about symptoms in this category. A, B, and D would likely all be part of the assessment and intervention plan, but only after a provisional diagnosis has been reached.

A social worker in a community agency receives a referral for a child with severe PTSD. The social worker has no training or experience treating children with PTSD. The social worker should: A. Attend a training on PTSD in children B. Conduct an initial assessment and make treatment recommendations C. Refer the child to appropriate services D. Treat the child after getting consultation

C. Refer the child to appropriate services

A social worker in a family service agency is working with an 8-year-old girl who discloses in session that she is being sexually molested by an older boy at school. As she is talking, the social worker realizes that the boy is in a therapy group that she facilitates at the school. The social worker should NEXT: A. Obtain more specific details about the abuse. B. Question the other client about the allegation after the next group. C. Report the abuse to the appropriate authorities. D. Contact the school principal to report the abuse.

C. Report the abuse to the appropriate authorities. RATIONALE: Child abuse has occurred, so a report is mandated. A and B is the role of the child abuse team. D could be done but C is legally required.

A hospital social worker meets with parents who refuse to provide appropriate medical care to their 6-year-old daughter with a life threatening illness because of their religious beliefs. The child is receiving some care within the guidelines of their church's ministry but the child's condition is getting worse. How should the social worker respond to this dilemma? A. Discuss the issue with the hospital staff. B. Not report the situation because the family's right to self-determination restricts the social worker's actions. C. Report the situation to Child Protective Services. D. Continue to strongly encourage the family to take a different action in the best interests of the child.

C. Report the situation to Child Protective Services. Medical Neglect

All of the following have been identified as community functions EXCEPT: A. Socialization B. Social Control C. Resilience D. Mutual support

C. Resilience RATIONALE: This is a factual recall questions regarding knowledge of community functions. Resilience is the only term that is not part of community functions.

During a utilization review phone call, a social worker is asked by the managed care representative to provide specific details of the sexual abuse incidents the client experienced. The social worker should FIRST: A. Provide all requested information to the reviewer B. Refuse to give specific information to protect the client's privacy C. Review the release of information with the client prior to providing information D. Review the managed care contract with the supervisor prior to providing information

C. Review the release of information with the client prior to providing information

A 38-year-old woman with Borderline Personality Disorder continually engages in scapegoating during a weekly treatment group. After consultation, the social worker facilitating the group decides to dismiss the client from the group due to the destructive nature of her in-group behaviors. What should the social worker to FIRST in order to facilitate termination? A. Inform the client during a group session that she will be leaving the group in 2 weeks. B. Ask the group whether the client should be dismissed from the group. C. Schedule an individual appointment with the client to discuss the decision. D. Refer the client for individual treatment before termination is discussed.

C. Schedule an individual appointment with the client to discuss the decision. RATIONALE: C is the best answer because this is a matter that should be handled privately and confidentially. A and B are inappropriate and would clearly shame the client in front of other group members. D could be done as part of the individual appointment, but would not be done

A 30-year-old man presents for treatment on the advice of his mother, who is concerned that he has not yet found someone with whom he can "settle down." The therapist notes during the assessment that the man fails to make eye contact and regards the therapist in a suspicious manner. When asked about his personal relationships, the man notes that he does not have any close friends because "no one likes a person who can read minds" and that he just doesn't "fit in" with his co-workers. What is the MOST likely diagnosis in this case? A. Paranoid Personality Disorder B. Schizophrenia C. Schizotypal Personality Disorder D. Schizoid Personality Disorder

C. Schizotypal Personality Disorder (a little odd)

A medical social worker starts a job at a hospital that has no referral strategy for the patients in need of social service. The social worker is invited to make a referral strategy. Which of the following methods BEST links social workers to those clients who need them most? A. Interviews of all new patients by the social worker. B. Visible and available social work office and staff. C. Team staff meeting, which include social workers, to discuss all current patients. D. Standardized referral forms for in-house physicians to use.

C. Team staff meeting, which include social workers, to discuss all current patients. RATIONALE: C is the best answer because it will foster a collaborative approach and ensure that social workers are aware of the needs of patients. A is too time intensive; B may not directly address the needs of patients. D may be helpful, but may not increase physicians' rate of referral or their ability to identify patients in need of social work services.

A 6-year-old child is brought to therapy by her foster parents. When the therapist enters the waiting room, the child runs up to her, wraps her arms around the therapist's legs and says, "Am I going home with you?" The foster parents are embarrassed by this behavior and say, "we've only had her for 2 weeks, but she does this with everyone!" What conclusion should the therapist draw from this behavior? A. The child may have Reactive Attachment Disorder, Inhibited Type B. The child has had positive relationships with adults and is securely attached C. The child may have Reactive Attachment Disorder, Disinhibited Type D. The child has most likely been abused

C. The child may have Reactive Attachment Disorder, Disinhibited Type

A family meets with a social worker for family therapy due to concerns regarding their 16-year-old daughter. The mother reports that their daughter was recently diagnosed with a medical condition that requires medication monitoring and a strict diet. She says that her daughter has many friends, and does well in school, however has recently been questioning her parent's authority and demanding to make her own choices. The father also reports that their daughter refuses to follow her required diet. This behavior is MOST likely due to? Response: A. Her parents misunderstanding of typical adolescent behavior. B. The client's use of illicit substances in addition to medication. C. The client's need to exert control over a part of her life. D. Noncompliance with medication in combination with irregular diet.

C. The client's need to exert control over a part of her life. Rationale: C is the best answer because the stem describes normal adolescent development. A may be true, but is not the reason for the daughter's behavior. B is not the best answer because there is nothing in the stem to indicate substance abuse. D is not the best answer because the stem does not mention noncompliance with meds, and while dietary changes may impact mood, they are not the MOST likely reason for the behavior.

A 15-year-old who was sexually abused for two years by a relative is meeting with a social worker for an initial evaluation. The social worker wants to assess the impact that the abuse has had on the client. To accomplish this, the social worker should focus on: A. The client's sexual identity. B. The client's level of repression. C. The client's psychosocial functioning. D. The client's self-care.

C. The client's psychosocial functioning. Rationale: C is the best answer because it most fully addresses the social worker's need to assess the impact of the abuse. While it is possible that A, B, and D may have all been affected, if the social worker focuses on any one of the three, he/she might miss something else. Understanding the client's current psychosocial functioning will give the social worker the best overall picture of how the abuse has impacted the client.

A social worker is developing a support group for adults who provide caregiving to their aging parents. The social worker wants to focus on coping skills to manage the stressors that accompany caregiving. In designing the group, the social worker should consider that caregiver stress is PRIMARILY linked with: Response: A. The duration of complicated hospitalizations. B. Whether or not the caregiver has a family of his/her own. C. The duration of the illness. D. The parent's age at onset of the illness.

C. The duration of the illness. Rationale: C is the best answer because duration of the illness is the factor most likely to produce stress in the caregiver. A, B, and D are not as likely to produce stress.

A hospital emergency room social worker meets with a 45-year-old man who recently immigrated from Mexico and has been experiencing severe panic attacks and anxiety. When the social worker approaches him, he says that he does not need any mental health treatment and that she can leave. The MOST likely explanation for his behavior is: A. His preference for seeking treatment from a curandero. B. His denial that he has a mental health problem. C. The negative cultural response associated with mental health treatment. D. A miscommunication due to a language barrier.

C. The negative cultural response associated with mental health treatment. Rationale: C is the best answer because many cultures associate stigma with mental health treatment. A is not the best answer because it makes an assumption that is not indicated by the information in the stem. B may be true, but is not as likely as C. D is not the best answer since the stem gives no indication that there is a language barrier between the client and social worker.

A man has been hospitalized for a month following a car accident that has resulted in him needing a wheelchair while he relearns to walk. The patient's elderly parents would like the client to move in with him so they can help him while he regains his mobility to which the client agrees. In developing discharge plans, the social worker should FIRST evaluate which of the following factors? A. Financial stability. B. Access to physical therapy and community resources. C. The parent's ability to provide adequate care to their son. D. The accessibility of the parents' home for a wheelchair.

C. The parent's ability to provide adequate care to their son. RATIONALE: It is most important that the social worker evaluate how well the parents would be able to care for their learn. Once that has been determined, the social worker can then assess other factors.

A diagnosis of Enuresis is appropriate when: A. The patient voids feces intentionally or unintentionally at last twice weekly. B. Medications may be causing the problem. C. Urine is unintentionally voided at least twice weekly and the patient is at least five years old. D. Urine is unintentionally voided at least twice weekly and the patient is at least six years old.

C. Urine is unintentionally voided at least twice weekly and the patient is at least five years old.

A pregnant 16-year-old and her mother seek consultation for the daughter's unplanned pregnancy. The mother expresses the desire to place the baby for adoption because she is in ill health and cannot help her daughter raise the baby. The social worker's FIRST response should be to: A. explore how they each feel about keeping the baby. B. help formulate an adoption plan. C. meet with the young woman alone to explore her feelings and wishes. D. suggest that the mother participate in the treatment sessions.

C. meet with the young woman alone to explore her feelings and wishes. Clients self determination

An 19-year-old single mother has a 22-month-old son placed in foster care for six months because of neglect. The mother has followed her reunification plan, and the foster care plan is to return the child to the home. The foster care social worker learns a week before the hearing that the mother is pregnant again. The social worker's BEST option is to request that: A. the child remain in foster care until after the birth of the baby. B. the mother regain legal and physical custody and close the case. C. physical custody be returned to the mother and the family situation be monitored by the agency for six months. D. the court date be postponed until the baby is born.

C. physical custody be returned to the mother and the family situation be monitored by the agency for six months. Always good to monitor after foster care.

You are called to evaluate a 64-year-old male with chronic obstructive pulmonary disease. He lives in an assisted living facility, and was brought to the emergency room by his daughter. She had taken him to lunch, and became distressed when he refused to return to the facility. He states he wants to live in his motor home, as he resents the loss of privacy at the facility. His daughter confirms he owns a working, fully self-contained motor home (i.e., stove, shower, refrigerator, etc). He has adequate funds. He plans to park the motor home in a nearby Kampgrounds of America (KOA) campground, where all utilities can be hooked up. He can have food and other supplies delivered. However, it is November and it is unseasonably cold. The doctor confirms that the patient is prone to pneumonia, and the daughter states "he will die if he doesn't return to the facility." The patient refuses to consider any other living situation. In this situation, the social worker should: a. Call the police and have them take the patient back to the facility. b. Call adult protective services for further intervention. c. Allow the patient to move into his motor home. d. Place the patient on an involuntary hold for suicidal behavior.

C: Allow the patient to move into his motor home. The patient has a plan sufficient to meet his needs for food, clothing, and shelter. He has the legal right to choose where he wishes to live, even if others are not comfortable with his choice. Calling the police will not help, as they cannot force him to return to the facility. Adult protective services may have a subsequent role, if the patient begins to exhibit marked self-neglect or cognitive changes, but they cannot force the patient either. Finally, the patient is not eligible for an involuntary hold, as he is not placing himself or others in danger based upon a diagnosable mental illness, intoxication, or other substance abuse. Careful collateral planning, however, will be important (ensuring the daughter visits and checks in on him, etc) to try and maximize his potential for success. After coping with the hardships of independent living, he may willingly return to assisted living.

18. The concepts of "pre-affiliation" (becoming acquainted), "power and control" (setting the roles), "intimacy" (developing cohesion), "differentiation" (independent opinion expression), and "separation" (moving to closure and termination) are all stages in: a. The lifecycle of a therapeutic relationship. b. General relationship cycles. c. Group development. d. Team cohesion.

C: Group development. During these stages, the social worker needs to: 1) facilitate familiarity and elicit participation; 2) clarify roles; 3) develop group cohesion; 4) support individual differences; and 5) foster independence. The use of a "Sociogram" (a chart or diagram depicting group member relationships) can aid the social worker in revealing, monitoring, and intervening (if necessary) in group member interactions and bonding.

4. In statistical research, a "Type I Error" (also called an "alpha error," or a "false positive") refers to: a. Failing to reject the null hypothesis when the null hypothesis is false. b. A failure to randomize research participants, thereby potentially introducing bias. c. Rejecting the null hypothesis when the null hypothesis is true. d. Assuming a normal statistical distribution when it is skewed.

C: Rejecting the null hypothesis when the null hypothesis is true. A failure to randomize research participants will potentially introduce bias, and may provide grounds upon which to invalidate a study, but it is not a type I error. Assuming a normal statistical distribution when it is skewed will violate the assumptions necessary to apply a proper statistical model to the analysis of data.

14. The following criteria are all used to distinguish substance abuse from substance dependence except: a. Symptoms of substance abuse are usually less severe than those of dependence. b. The problematic effects of abuse are usually limited to family, finances, employment, and legal issues (e.g., driving under the influence), while dependence also involves significant physiological problems. c. Substance abuse typically involves narcotics, while dependence typically involves non-narcotic drugs. d. Abuse is typically limited to recreational use, while dependence involves the need for increasing doses for the desired effect and withdrawal symptoms of not used regularly.

C: Substance abuse typically involves narcotics, while dependence typically involves non-narcotic drugs. In general, narcotics are more addictive than non-narcotic substances. Even relatively "benign" substances of abuse, such as marijuana, can produce dependence if used often enough, and mounting clinical and preclinical evidence suggests that a marijuana withdrawal syndrome also exists (however, it is not yet in the DSM). Common treatment medications include: Antabuse (disulfiram; for alcohol abuse); ReVia (naltrexone; for alcohol and narcotics); and Trexan (naltrexone; for alcohol and opioid dependence).

Second stage of community building where members vent frustration and disagreements and differences cannot be ignored

Chaos

- birth to puberty where childhood has 2 substages - archaic stage where there is sporadic consciousness -monarchic stage where beginning of logical and abstract thinking is - ego starts to develop

Childhood stage (carl jung)

- this theory identifies range of concepts to describe how people, groups, communities exchange information and how messages are conveyed and received

Communication theory

Ability to communicate is central to effective social work practice and has many forms - language, speech, written word

Communication theory

What should an infant be doing at 10 months?

Crawl, exhibit stranger shyness/anxiety

Parents of a 12-year-old with depression come to a session with their social worker after their child's psychiatrist has recommended an antidepressant. They tell the social worker that they do not want to give their child medication and want to continue having the child see the social worker for therapy as the only mode of treatment. The social worker believes that the child's depression will not respond to therapy alone and shares this with the parents, but the parents continue to refuse to consider medication. What should the social worker do NEXT? A. Encourage the psychiatrist to intervene with the parents. B. Refer the family to a different social worker due to a value conflict. C. Provide treatment to the daughter without the use of medication. D. Address the positives and negatives of not using medication with the parents.

D. Address the positives and negatives of not using medication with the parents. RATIONALE: The social worker should provide information about the positives and negatives of therapy so the parents can have a fully informed decision. A was already tried, B would be unnecessary and C could be done after D.

A therapist is called to the emergency room to evaluate a 25-year-old man who was brought in by police for public indecency. The therapist attempts to interview the man but has difficulty due to his rapid, tangential speech and his refusal to answer questions because he fears the therapist will "pass the info along to the wrong people." The client adds that he has been awake for three days. The therapist notes that he is sweating profusely. A likely diagnosis is: A. Opioid Intoxication B. Alcohol Intoxication C. Cocaine Intoxication D. Amphetamine Intoxication

D. Amphetamine Intoxication (affects for 3-4 days - Rocks) A. Opioid Intoxication (veg you out) B. Alcohol Intoxication C. Cocaine Intoxication (stimulant/only stay up for 1 day)

A 45-year-old client meets with a social worker due to a long history of prescription drug abuse. During the intake assessment, the client states that he has been in many treatment programs, but has been unable to stay sober for longer than a few months. The client says that he doesn't think anyone can help him anymore and is afraid that he is going to turn into his father, who passed away after overdosing on drugs when the client was a teenager. According to the relapse prevention model, what should the social worker do FIRST? A. Refer the client to Narcotics Anonymous. B. Explain the importance of involving the client's family in the recovery process. C. Discuss the genetic and environmental impact of his father's addiction. D. Ask clarifying questions to identify the client's motivation to change his behavior.

D. Ask clarifying questions to identify the client's motivation to change his behavior. Rationale: D is the best answer because motivation for change is a significant part of the relapse prevention model and will help dictate the course of intervention. A, B, and C do not relate specifically to a relapse prevention model of treatment.

A social worker is treating a mother and her 15-year-old son for frequent conflicts. At the end of one session, the mother points out how helpful her son has been around the house; the son replies by saying, "I didn't really do that much." What should the social worker do FIRST? A. Point out the son's inability to accept praise from his mother. B. Ask the mother why she's complimenting the son now. C. Suggest that perhaps the son resents having to help around the house. D. Assist the son in accepting his mother's praise.

D. Assist the son in accepting his mother's praise. Rationale: D is the best answer because it directly addresses the dynamic that has just occurred and uses a positive strategy to create change. A, B, and C might all be good interventions at a future session, but would not be done before working to help the son accept the mother's praise in the moment.

A court-ordered client with a history of violence shares that he is really pissed off at his former boss and is "going wait for him after work on Friday to make him pay." When the social worker asks the client what he means by the statement, the client tells the social worker not to worry about it. How should the social worker proceed? A. Document the exchange in notes. B. Inform the court of concerns. C. Warn the boss and inform the police. D. Attempt to clarify the statement again.

D. Attempt to clarify the statement again.

A mother meets with a social worker due to concerns with her seven-year-old son. The mother reports that her family recently survived an earthquake that destroyed their house and the surrounding area. She is worried about her son because any time he hears a loud noise he starts to cry inconsolably and won't leave her side. In behavioral therapy, what is this an example of? A. Extinction B. Negative reinforcement C. Operant conditioning D. Classical conditioning

D. Classical conditioning Rationale: This is a recall question: the stem describes an example of classical conditioning in which the client develops a conditioned response (crying and clinging to mom) when exposed to stimuli (loud noises) that represent a previously experienced event (the earthquake). A, B, and C would all be described differently.

A hospital-based social worker receives outcome data that indicates a large percentage of geriatric patients with knee replacements are functioning at lower levels one year after surgery. As part of the medical team's response, the social worker is asked to develop a psychosocial program that will enhance functioning. The social worker's BEST approach is to: A. Provide pre-operation workshops to highlight challenges. B. Coordinate the group with the medical director. C. Recommend additional physical rehabilitation. D. Create a drop-in support group.

D. Create a drop-in support group. RATIONALE: Since the question ask about a psychosocial program, C does not make sense since it refers to physical rehab; A does not address the patients' functioning after surgery and B is an incomplete answer.

A social worker in a community clinic meets with a woman who presents with issues related to her job. She discusses a recent incident in which she was gone from work for several days without notifying her boss or coworkers, although she has no memory of this. The client has found a pamphlet and tickets from a Broadway show in her home, although she states that she's never been to New York. The client is describing which of the following conditions? A. Depersonalization B. Derealization C. Delirium D. Dissociation

D. Dissociation RATIONALE: D is the best answer because engaging in activities with no consciousness or memory of the activities is a form of dissociation; A, B and C describe different phenomena.

A school social worker is facilitating a meeting with students at a high school after a classmate committed suicide over the weekend. The social worker should FIRST: A. Normalize the grief process. B. Provide referrals to mental health counseling. C. Find out if there were any signs that were missed. D. Encourage students to express their reactions.

D. Encourage students to express their reactions.

A community-based social worker is counseling a client who tested positive for HIV five months ago. The client admits to not informing sexual partners of the diagnosis. The social worker should FIRST: A. Discuss AIDS prevention activities with the client. B. Inform the client's partners of the positive test. C. Contact the local public health authority to discuss the issue. D. Encourage the client to inform sexual partners.

D. Encourage the client to inform sexual partners.

A social worker meets with a couple who have a two-year-old and a newborn child. The mother says she feels like she is doing all the work around the house by cooking, cleaning, and taking care of the children. Her spouse says that she never makes time for him and it feels like she would rather be with the kids than with him. The couple MOST likely is: A. Endangering their children. B. In need of professional home-based support services. C. Using their children to mask deeper problems in their relationship. D. Experiencing stress associated with a life cycle change.

D. Experiencing stress associated with a life cycle change. Rationale: D is the best answer because the stem described a very typical pattern in families experiencing a life cycle change (new baby). A makes the assumption that there is a safety issue, when nothing in the stem indicates that. B is not supported by the stem, since the couple's problem, while very real, is not unusual. C is not supported by the information in the stem.

A social worker meets with a college freshmen who was recently diagnosed with schizophrenia, and her parents. The girl is on medication, which has helped, however, her thoughts have become impaired and she has difficulty concentrating. In family sessions, the parents focus on her goal of becoming a surgeon like her father. The social worker should NEXT: A. Identify the steps to reaching this goal. B. Refer the girl to a support group for individuals with schizophrenia. C. Ask the girl if that is what she wants. D. Have a separate meeting with the parents from the young woman.

D. Have a separate meeting with the parents from the young woman. RATIONALE: The parents clearly have unreasonable expectations. It is important to address this with them privately and process their feelings about it. C could be done later.

A school therapist receives a referral for a 10-year-old boy for inattention and poor grades. During observations and interactions with the student, the therapist experiences the boy as intelligent and highly verbal but notes that he has difficulty following instructions and often leaves assignments only partially completed. The boy's school records indicate that he was an above average student until the 3rd grade. The initial diagnosis is MOST likely A. Conduct Disorder B. Attention-Deficit/Hyperactivity Disorder, Predominantly Inattentive Type C. Asperger's Disorder D. Learning Disorder

D. Learning Disorder

A mother brings her 16-year-old son to the emergency room stating that she does not know what is wrong with him. She reports that starting a few weeks ago, her son has been forgetful about his homework and chores around the house, has fallen down the stairs twice, and this evening attempted to make an expensive purchase from an infomercial. When asked, the mother reports that the only other change in the teen's life is that a new neighbor moved in a month ago. The MOST likely cause for the son's behavior is: A. Peer conflict B. Sleep loss C. Typical adolescent behavior D. Marijuana abuse

D. Marijuana abuse Rationale: This is a recall question; D is the best answer because the behaviors described in the stem are most associated with marijuana abuse. A is not the best answer because no social problems are indicated in the stem. B is not the best answer because it is not as likely as D and would be described differently in terms of the boy's behavior. C does not make sense given the significant changes in the boy's behavior.

A social worker at a non-profit agency is assigned to be an advocate for a group of refugees who have recently been relocated. The refugees are in need of food, clothing, basic medical care. They do not speak English. In order to be successful, what should the social worker do FIRST? A. Provide supervisory direction to the refugees about how to obtain services B. Consult with colleagues about what services may best meet the refugees needs C. Review agency policy regarding available services and their requirements D. Obtain additional information about this client group and their specific needs

D. Obtain additional information about this client group and their specific needs

A social worker begins to notice a pattern of interaction between the client and the social worker. The social worker offers specific suggestions to help the client achieve personal goals and the client thanks the social worker and then identifies the reasons why each suggestion will not work. To promote progress, the social work should FIRST: A. Help the client identify this self-defeating attitude. B. Remind the client of the treatment goals. C. Focus conversations on the client's strengths and motivation. D. Point out this pattern to the client.

D. Point out this pattern to the client. RATIONALE: D is the best option in this case because the client may not be aware of the pattern; simply by sharing the observation of this pattern, the therapist may be able to help the client change. A, B and C may be viable interventions, but not before the observation of the pattern is shared.

A client is admitted to an emergency room with a broken jaw. She claims that she fell down the stairs and doesn't want to talk about it. Her boyfriend shows up an hour later, brings her flowers, and then leaves for work. After completing the initial assessment, the social worker should: A. Confront the client about the denial of domestic violence. B. Provide psychoeducation about the cycle of violence. C. Work with the client to make a safety plan. D. Provide the client a referral to domestic violence shelter.

D. Provide the client a referral to domestic violence shelter. (she said she doesn't want to talk about it)

A social worker is meeting with a client at a community mental health clinic. During the session, the client states that he is currently unemployed and can't afford the sliding scale fee that was predetermined. What should the social worker do FIRST? A. Speak with a supervisor to address providing a lower fee for the client based on need. B. Find out what the client feels comfortable paying. C. Explore other areas client can save money to afford therapy. D. Provide the client with the agency's policy regarding inability to pay. RATIONALE: It is important to take into account the setting. D is the best place to start and would determine what would come next.

D. Provide the client with the agency's policy regarding inability to pay. RATIONALE: It is important to take into account the setting. D is the best place to start and would determine what would come next.

When discussing the fee with a potential new client the social worker learns that the client recently lost her job and is living off her savings. Even after sliding the fee to the lowest amount, the client states that she only thinks she can afford four or five sessions. The social worker should: A. Agree to see her at the reduced rate. B. Lower the rate to accommodate her situation. C. Find out what issues the client hopes to deal with in therapy. D. Refer the client to a low-fee mental health services.

D. Refer the client to a low-fee mental health services. RATIONALE: Since no relationship has been established with the client yet, the social worker should provide the client with referrals to a low-fee mental health agency (D). A and B would be incorrect because four or five sessions at a reduced rate would most likely not be effective for the client. C is incorrect because it doesn't address the client's inability to pay for therapy.

A social worker in a nursing home is asked to meet with an 89-year-old man who is in poor health but continues to be mentally sound. The man has begun to refuse meals and the doctors have told him that he will soon require a gastric feeding tube in order to obtain nourishment. The man's son has been trying to talk his father into this intervention, but the man refuses. What should the social worker do FIRST? A. Make a referral for family therapy for the father and son. B. Assist the son in working through his feelings about his father's health. C. Provide the son with information about the father's right to refuse treatment. D. Refer the man for a competency evaluation.

D. Refer the man for a competency evaluation. RATIONALE: The first thing to determine is whether the father is competent to make the decision to refuse the needed feeding tube. Once that is determined, A, B, or C could be done.

A social worker has seen a young adult client for a few months. During a session, the client tells a social worker that he has recently participated in a violent criminal act and plans to repeat the offense. The social worker should: A. Discuss alternative ways the client can express his aggression. B. Maintain client confidentiality. C. Obtain a written contract that the client will not act on thoughts. D. Report the intent according to jurisdictional law.

D. Report the intent according to jurisdictional law.

A college counselor starts brief treatment with a senior who was referred by his honors thesis advisor. The student denies any need for treatment but engages in the intake, telling the therapist that he prefers to be alone and rarely goes home to see his family. He reports confusion around what his peers do on the weekends and denies any interest in romantic or sexual relationships. He tells the therapist that he's had the same roommate since freshman year but does not really know much about him. What is the MOST likely diagnosis? A. Schizotypal Personality Disorder B. Antisocial Personality Disorder C. Avoidant Personality Disorder D. Schizoid Personality Disorder

D. Schizoid Personality Disorder (someone who likes to be alone)

A client is referred to a therapist after being discharged from the psychiatric hospital. The records indicate that he has been hearing voices for the past four months and he believes he is being followed. According to the DSM-IV-TR criteria, the MOST likely diagnosis for the client is: A. Brief Psychotic Disorder B. Schizophrenia C. Schizoaffective Disorder D. Schizophreniform Disorder

D. Schizophreniform Disorder (1-6 months) Brief Psychotic Disorder (under a month) Schizophrenia (at least 6 months) Schizoaffective Disorder (must have mood disorder)

Clients discharged from a local psychiatric hospital all receive monthly medication monitoring and supportive case management from a mental health center. Despite these services, the hospital social worker notices that many of these clients experience frequent crises after discharge and need to be re-hospitalized. What is the MOST likely reason for this? A. Client's families are not engaged in the discharge planning process. B. Hospitalization did not address the clients' need for coping skills. C. The program staff are overworked and unable to provide clients with individual attention. D. Sufficient follow up services are not available in the community.

D. Sufficient follow up services are not available in the community. Rationale: D is the best answer because lack of community services is the MOST frequent reason for patient's re-hospitalization. A makes an assumption that the patients' all have families; B and C both assume that the hospital is not addressing the patients' needs, when nothing in the stem would indicate this.

A client who has recently immigrated to the United States seeks therapy to adjust to the client's culturally different spouse. To assess the client's level of acculturation, which of the following factors should the social worker take into consideration FIRST? A. Newly immigrated individuals are often hesitant to challenge cultural traditions in their marital relationships. B. Newly immigrated individuals are resistant to learning the rules of a new country. C. Gender differences cause more discomfort than acculturation issues. D. The pace of acculturation may vary from group to group.

D. The pace of acculturation may vary from group to group. Rationale: D is the best answer because it does not make assumptions and encourages the social worker to understand more about this individual's experience. A and B both make negative assumptions. C is not true.

A social worker has been seeing a client intermittently for two years, and they have a strong relationship. The client is often fearful and suspicious of new situations. The focus of interventions has been to promote client independence and personal decision-making. The client recently learned of an opportunity to move into a group home that offers better social support than the current living arrangement. The client is reluctant to discuss the new housing option and reports being worried about making a change. The social worker asks the client to tour the home and see what it is like before rejecting it. Because he trusts the social worker, the client agrees. This is an example of: A. The usefulness of countertransference in the therapeutic relationship. B. Compartmentalizing the client's concerns to make them more manageable. C. Empowering the client throughout the process of self-determination. D. Using the social worker/client relationship as an intervention tool.

D. Using the social worker/client relationship as an intervention tool.

10. All but one of the following are National Association of Social Workers (NASW) standards for cultural competence: a. Social workers should endeavor to seek out, employ, and retain employees who provide diversity in the profession. b. Social workers shall endeavor to resources and services in the native language of those they serve, including the use of translated materials and interpreters. c. Social workers should develop the skills to work with clients in culturally competent ways, and with respect for diversity. d. Social workers should work with diverse clients only if they have had specific training in that client's unique cultural background.

D: Although it is ideal for social workers to receive specific training regarding each of the individual minority populations they typically serve, they should still ensure that someone from an unfamiliar background receives needed services even where no staff with special training in that background is available.

15. Name the four classic diagnostic "A's" of schizophrenia: a. Awareness, ambivalence, autism, and associations. b. Agitation, awareness, associations, and autism. c. Affect, anxiety, ambivalence, and awareness. d. Affect, associations, ambivalence, and autism.

D: Affect, associations, ambivalence, and autism. In 1911, Eugen Bleuler coined the term schizophrenia, and defined it using his now-classic four "As:" Affect (blunted emotional response to stimuli); associations (loosening, disordered thought patterns), ambivalence (an inability to make decisions due to poor information integration and processing), and autism (a preoccupation with the self and one's thoughts). Common medications for treatment: Clorazil (clozapine), Haldol (haloperidol), Loxitane (loxapine), Mellaril (thioridazine), Prolixin (fluphenazine), Risperdal (risperidone), Stelazine (trifluoperazine), Thorazine (chlorpromazine), and Zyprexa (olanzapine).

7. An early cognitive theorist, who worked directly with Freud, established a theoretical orientation that differed from Freud's in three key features: 1) an individual's personality is best perceived as a whole, rather than as having hierarchical segments or parts; 2) social relationships drive behavior more than sexual motivations; and 3) current beliefs and thoughts play a far greater role in human behavior than is suggested via psychoanalytic theory, which is based largely in the unconscious and in past experiences and beliefs. The name of this theorist is: a. Lawrence Kohlberg. b. Anna Freud. c. Albert Ellis. d. Alfred Adler.

D: Alfred Adler. Adlerian theory also includes a biological view, largely absent in Psychoanalytic Theory, recognizing that hormonal changes, physical illness, chemical imbalances, and neurological disorders can dramatically influence capacity and behavior. It is important to note, however, that Alder still locates false beliefs, irrational thoughts, and misconceptions in the unconscious mind.

9. You have been contacted by a couple to assist them with issues of marital discord. They have been married about six months. The wife presents as vulnerable, tearful, and anxious, and the husband presents as angry and overwhelmed. The wife openly claims that "he has never loved me," and expresses anger that he married her without "the proper feelings." The husband responds that he has "done everything possible" to "prove" his love (to the point of near bankruptcy and jeopardizing his employment with frequent absences), but nothing is sufficient. During the interview, you discover that she has had many short-term relationships in the past, that she has a history of suicide gestures and "fits of rage." Further, she frequently demands a divorce and then begs him to stay, is routinely physically assaultive, etc. The most likely diagnosis is: a. Intermittent explosive disorder. b. Histrionic personality disorder. c. Paranoid personality disorder. d. Borderline personality disorder.

D: Borderline personality disorder. Individuals with this diagnosis will exhibit: frantic efforts to avoid real or imagined abandonment; unstable and intense interpersonal relationships (especially extremes of idealization and devaluation); an unstable sense of self; extreme impulsivity (e.g., spending, sex, drug use, reckless driving, binge eating, etc); recurring suicidal behavior (gestures or threats, or self-mutilating behavior); affective instability due to reactivity of mood; chronic feelings of emptiness; intense anger (e.g., frequent displays of temper, recurrent physical fights); transient, stress-related paranoid ideation; or severe dissociative symptoms. However, as with all Axis II disorders, this diagnosis cannot properly be made during a first contact, but must be substantiated over a course of clinical contacts sufficient to compel the diagnosis to be made (DSM-IV).

19. When a client seems overwhelmed or uncertain how to share further, it can help to break down the concerns at hand into smaller, more manageable parts. This communication technique is known as: a. Fragmentation. b. Sequestration. c. Downsizing. d. Partialization.

D: Partialization. For example, "Well, if we take these things one at a time, maybe we can start with..."

6. Self-Psychology, as postulated by Heinz Kohut, acknowledges that personality is partly formed by social structure. A cohesive self is achieved by incorporating the perceptions and functions of healthy significant others and objects into an internalized self structure through a process called: a. Empathic mirroring. b. Rapprochement. c. Differentiation. d. Transmuting internalization.

D: Transmuting internalization. Empathic mirroring is the process by which the mother demonstrates ("reflects") care and understanding of the child, in turn helping the child to develop a self-identity. Rapprochement is a term from object relations theory, indicating the need for an infant to seek independence while still retaining security. Differentiation is a substage in object relations theory, where an infant begins to look at the outside world, as opposed to the inward focus common to infants younger than five months of age.

A 30-year-old gay male is seeking treatment in a mental health agency. The intake social worker begins to gather information to determine this individual's qualification for services. The client states that he has been feeling sad and hopeless, and is losing interest in activities in which he used to participate. He states that he and his partner have recently broken up and that he has been spending less time with his parents and siblings. The intake worker should refer this individual for treatment and identify his presenting problem as His Homosexuality Relationship issues Family issues Depression

Depression

Older individuals are often misdiagnosed with Alzheimer's when, in fact, the more accurate diagnosis is Anxiety Depression Thyroid Disorder Delirium

Depression. Approximately 55% of the elderly suffer from depression and are often misdiagnosed with Alzheimer's or Dementia. Anxiety (answer #1) is not correct, although anxiety can be a problem in older adults; anxiety is not a characteristic that would be a symptom of Alzheimer's. Thyroid disorder (answer #3) is not correct because this condition is a malfunction of the thyroid gland, resulting in a variety of medical symptoms, usually not similar to symptoms of Alzheimer's. Delirium (answer #4) is not correct, as Delirium is characterized by a disturbance of consciousness and a change in cognition that develop over a short period of time.

3. During a first interview, a young married woman, who has recently given birth to her first child, complains of loss of appetite and inability to sleep. She seems somewhat disheveled and disorganized, displays a mournful demeanor, and says she feels unmotivated and overworked. The social worker's first activity would be to: A) Assess the suicidal risk B) Determine the frequency and duration symptoms C) Record a thorough history D) Provide reassurance now that the client has sought treatment

Determine the frequency and duration of symptoms

A concept where person has ability for family member to tell difference between their experience's and identify themselves from others

Differentiation of self

lack of coherent attachment strategy, child showing freezing, rocking, similar bxs upon return of caregiver - caregiver maltreatment, role confusion, abuse

Disorganized

separation of postponement of a feeling that normally would accompany a situation or thought

Dissocation

Constant interaction with environment and each system impacts one another

Ecological systems theory

unhealthy way of dealing with family growing up or building one's own individual identity

Emotional cut off

Third stage of community building where members embrace need of group and balance

Emptiness

A social worker has recently learned from the wife of a couple he is seeing that the wife believes that she is a lesbian. She states she does not want to share this with her husband. During the next session, the husband asks his wife if she likes women better than men. The wife answers "No." What is the FIRST response that the social worker should make in this situation? Encourage the wife to be honest Encourage the husband to ask the wife after the session Say nothing but plan on speaking with the wife after the session Encourage the couple to discuss the issue

Encourage the couple to discuss the issue. The responsibility for establishing a relationship of honesty rests with the couple. The social worker can facilitate the process of developing such a relationship by encouraging dialogue about this and other important issues. Encouraging the wife to be honest (answer #1) is not the best answer for this scenario, as the social worker would be indicating that he has information that the husband does not have. The social worker may, at another time, need to help the wife decide how she will handle this information with her husband. Encouraging the husband to ask the wife after the session (answer #2) and saying nothing but planning on speaking with the wife after the session (answer #3) are both incorrect answers because these would be interfering with the couple dealing with their own situation.

- early childhood - focused on sense of personal control - toilet training is vital for independence - control over food choices, food preferences, clothing, etc. - success leads to confidence

Erikson's Autonomy/Shame and doubt stage

- adulthood - career and family - feel like contributing to the world by being active in home/community - develop care in this stage - proud of your accomplishments, unity w/partner, watching children grow up

Erikson's Generativity/Stagnation Stage

- children explore independence and sense of self - receiving proper encouragement and reinforcement w/personal exploration emerge w/strong sense of self and feeling independent and control - completing this stage leads to fidelity and ability to live by society standards

Erikson's Identity/Confusion stage

- early school (age 5-11) - through social interactions can develop sense of pride in their accomplishments and abilities - if encouraged and commended by parents/teachers will feel competent and belief in skills

Erikson's Industry/Inferiority stage

- preschool years - assert power and control over the world through directing play and social interactions - success leads to child able to lead others and have a sense of purpose - failure leads to guilt, self-doubt and lack of iniative

Erikson's Initiative/Guilt stage

- old age - reflecting back on life - feels like life was wasted and regrets - bitterness /despair - but if proud of life will feel integrity

Erikson's Integrity/Despair stage

- adulthood - exploring personal relationships - develop close/vital relationships which leads to committed, secure - strong sense of personal identity important to develop interpersonal relationships - success leads to virtue of love, forming lasting relationships

Erikson's Intimacy/Isolation stage

- Birth to 1 year old - trust comes from caregivers and quality of care - trust happens and child feels safe and secure - mistrust happens and child feels inconsistent, insecure, emotionally unavailable

Erikson's trust/mistrust stage

A social worker receives a new client who is seeking cognitive behavioral therapy, which is not the social worker's treatment of expertise. The social worker should Attempt this intervention Refer the client to an expert Explore the client's reasons for wanting cognitive behavioral therapy Use his own techniques as he or she is well trained to meet the client's needs

Explore the client's reasons for wanting cognitive behavioral therapy. As the social worker discusses this with the client, it may become evident that the social worker is able to provide the services the client desires. Alternatively, the discussion may reveal the need to refer the client elsewhere. Attempting this intervention (answer #1) is not correct because a social worker should be trained and competent in the interventions he or she uses. Referring the client to an expert (answer #2) is not the best answer because the social worker should find out more about the client's request before referring the client to an expert in cognitive behavioral therapy. Using the social worker's own techniques as he or she is well trained to meet the client's needs (answer #4) is not correct because the client has asked specifically for cognitive behavioral therapy and should be part of the decision-making process regarding whether other techniques could be as beneficial.

4. When one individual in a family markedly improves another simultaneously deteriorates this would be recognized as an example of A) Role complementarily B) Pseudomutuality C) Family homeostasis D) Family fusion

Family homeostasis

Family affects individual and family functioning across life span

Family systems

First stage of group development where you become oriented to goals and procedures of the group. Info and structure are critical

Forming

- Puberty to death stage - sexual interest of opposite sex - interest in welfare of others

Freud's Genital stage

- 6 years old to puberty - libido interests suppressed - development of ego and superego can help calm these interests - exploration of peer relationships and social interactions

Freud's Latent stage

- discovering difference in boys and girls (genitals) - 3 to 6 years old - conflict can be castration anxiety as mother's attention is sought

Freud's Phalic stage

- 1 to 3 years old - controlling bladder and bowels - conflict in potty training - learn to control body needs which leads to accomplishment and independence

Freud's anal stage

- birth to 12 months - primary interaction through mouth - rooting sucking is important - conflict is weaning

Freud's oral stage

Holistic approach to look at an individual's systems and how they interact with each other

General Systems Theory

A 65-year-old Hispanic-looking woman walks into a clinic for her first therapy session. The social worker is a bilingual therapist. Under these circumstances, what is the FIRST thing that the social worker should do? Greet the woman in Spanish to let her know that the social worker is respectful of her culture Ask the woman if she would like to speak to the social worker in Spanish Consult with the state to see if English is the official language and inquire about related laws Greet the woman in English

Greet the woman in English

What should an infant be doing at 3 months?

Hold up their head for prolonged period of time, make vowel noises, follow objects

If you have the condition where P = .05 or .01, you as a researcher should: a. Reject the null hypothesis b. Accept the null hypothesis c. Reassess the study d. Perform a post test

If you have the condition where P = .05 or .01, you as a researcher should: a. Reject the null hypothesis

A client was recently diagnosed with Multiple Sclerosis. She immediately began to learn everything she could about the illness: reading books, journal articles, and the latest experimental research. The client's response to her diagnosis is representative of which defense mechanism? Intellectualization Regression Sublimation Repression

Intellectualization

6 I's in community development Introduction Interaction Involvement Influence Investment Identity

Introduction - welcoming, orienting, teaching Interaction - sharing common experiences Involvement - everyone is important and needed Influence Investment - interaction is gentle and there is open honest communication Identity - there is a "we" and "us" / unity and purpose

At 8-12 months the child will have arms and hands that are more developed that the feet and legs, with the legs possibly appearing bowed, and the feet appearing flat.

It is at this age that the child will begin to explore, and has developed the ability to crawl and manipulate objects.

At 12-24 months the child is deemed a toddler. At this age the rate of growth will slow, and the head growth rate will slow dramatically to about 1 inch every 6 months.

It is at this developmental stage that the chest circumference will grow larger than that of the head.

Physiological: hunger, thirst, bodily comforts, etc.

Maslow's 1st Hierarchy of Need

Safety and security: out of danger

Maslow's 2nd Hierarchy of Need

Belongingness and Love: affiliate with others, be accepted

Maslow's 3rd Hierarchy of Need

Esteem: to achieve, be competent, gain approval, and recognition

Maslow's 4th Hierarchy of Need

Type of addiction model where addiction acts as a "brain disease", there is neurotransmitter imbalance. The agent is the drug, the vector is the dealer and the host is the addict. They think "stamp out" disease by eliminating drugs.

Medical Model

Type of addiction model where addicts are "weak" and can overcome a compulsion to use with willpower. Drug abusers choose to use drugs. Drug abusers are anti-social and should be punished. Drugs are evil.

Moral model

Fifth stage of group development where there is not functioning as group much longer and you work on closure

Mourning

A 60-year-old female client has been meeting with a social worker to discuss dissatisfaction in her marriage. Over the course of the first few sessions, the social worker noticed that the client spent little time discussing substantive marital issues and instead discussed her accomplishments, even when it did not fit into the conversation. She also talked about how she felt that it was important for her to spend her time with other well-educated individuals. The client described work situations in which she became angry and then tearful when not validated for her achievements. She also tried to get a manager fired for not implementing her ideas. The most likely diagnosis for this client would fall into which category? Cluster A Cluster B Cluster C None of the above

Narcissistic Personality Disorder- a Cluster B classification- is characterized by many of the behaviors the client exhibits. Cluster B Personality Disorders are evidenced by dramatic, erratic behaviors and include Histrionic, Narcissistic, Antisocial, and Borderline Personality Disorders. Cluster A Personality Disorders (answer #1) is not correct because those disorders are considered to be marked by odd, eccentric behavior and can result in diagnoses such as Paranoid, Schizoid, and Schizotypal Personality Disorders. Cluster C Personality Disorders (answer #3) is not correct because those disorders are distinguished by the anxious, fearful behavior commonly seen in Obsessive-Compulsive, Avoidant and Dependent Personality Disorders.

Third stage of group development where there is cooperation and team building.

Norming

Mutual emotional force within a family which occurs when there is a chronic family tension

Nuclear family emotional process

8. In a beginning interview with a client, the purpose of asking questions is to: A) Probe the client's unconscious feelings B) Obtain information about the presenting problem C) Direct and focus the client's conversation on their personal and family history D) Find out if the client really wants help

Obtain information about the presenting problem

Physical development: - 4 to 8 months child's head and chest circumference should be about equal. The head should continue to grow, at a rate of about ¾ inch per month until 6 months of age, and about 1/3 inch for each month thereafter. It is during this time period that the first evidence of teeth will begin to appear. Normally the incisors will come in first. It is this period that is associated with teething in our society.

On the motor skill development side, at this age the child should be able to pass an object from hand to hand, and should be able to grasp a bottle.

Fourth stage of group development where there is functioning to achieve group goals.

Performing

Understanding an individual and their bx in light of environmental contexts in which they live and act

Person In Environment

A teacher has noticed some dramatic behavioral changes in a 7-year-old boy over the recent weeks and has referred him to the school social worker. The child appears distracted in class, no longer finishes his daily assignments, has become aggressive toward peers, has soiled his pants on two occasions, and appears apprehensive around adults. Which of the following might likely be the cause of this child's behaviors? Possible neglect at home Possible sexual abuse Onset of Conduct Disorder Recent death of a loved one

Possible sexual abuse

10. He's having trouble adjusting to his wife's death, his work responsibilities, and caring for his daughters. These are viewed as the A) Dynamics B) Presenting problem C) Diagnosis D) Treatment Modality E) Values and Ethics

Presenting problem

Attributing a painful thought or idea to another person

Projection

First stage of community building where everything is "fine" and everyone gets along to avoid conflict

Psuedocommunity

Type of addiction model that looks as drug abusers as "self-medicating" and that drug abuse is a symptom of underlying psychological problems. Drug is a maladaptive coping strategy. They need to resolve inner conflict and then drugs won't be necessary.

Psychodynamic model

Attempting to provide a logical / rational explanation for something to avoid guilt or shame

Rationalization

7. The school social worker initiates an interview with Marie's family because of her poor grades. Now in fourth grade, Marie seems motivated and presents no behavioral problems. She lives in an intact, working-class family; the discussion with Marie's father makes it clear that he feels humiliated about her academic problems. During the interview, the mother says little. When she speaks, the father interrupts her. How should the worker first proceed? A) Make a report to the child protective services B) Note that Marie's performance problems are a reflection of the school's failure C) Reassure the parents that Marie's performance problems in school can be helped through cooperative effort by both the school and the family D) Refer the family for family treatment and parenting workshops

Reassure the parents that Marie's performance problems in school can be helped

What should an infant be doing at 2 years old?

Run or walk up stairs

What should an infant be doing at 6 months?

Saying multiple syllable sounds such as mama, dada; sit up by themselves

A 55-year-old male has been meeting with a social worker to address his alcohol dependence. Over the past few weeks he has cut down on his drinking. His arrives at his scheduled appointment sweating. He states he has been vomiting and he has a noticeable tremor in his hands. How should the social worker respond? a. Call 911, even if the client refuses to consent, as the client is likely detoxing and may experience seizures or possibly death. b. Encourage the client to decrease his alcohol intake more slowly as he is less likely to suffer such serious withdrawal symptoms. c. Talk to the client about the possible dangers of alcohol withdrawal and allow the client to make his own decision about whether or not to seek medical help. d. Encourage the client to withstand the uncomfortable symptoms for a few days as they will likely pass after the painful process of detox.

c. Talk to the client about the possible dangers of alcohol withdrawal and allow the client to make his own decision about whether or not to seek medical help. --Unless there's a medical emergency, the proper course is to explain the possible risks to the client and discuss whether or not he wants to seek medical treatment.

7. The primary role of a social worker is: a. building rapport b. providing support systems c. acting as a resource d. curing emotional or mental dysfunction

c. acting as a resource

What should an infant be doing at 12-20 weeks?

Show hand regard - recognize their hand and follow it with their eyes

Type of addiction model where the drug use is a learned behavior and drugs are used because they were modeled. Use drugs due to peer pressure. Environment leads to drug use (advertising). Drug use is a maladaptive relationship negotiation strategy.

Social Model

6. In facilitating a hospital support group for bereaved parents, a social worker can discuss a personal experience of a death of a child to: a. create empathy with the participants b. receive help from the group c. create a sense of shared experience d. establish role as expert on grief issues

c. create a sense of shared experience

What should an infant be doing at 1 year old?

Stand by holding on to supports, able to stand unsupported

Second stage of group development where there is resistance and challenging of leadership

Storming

A social worker is interested in conducting a study of drug use among adults of different ethnic backgrounds. Which of the following sampling strategies is most apt to yield a sample that reflects the ethnic composition of the area? Simple random sampling Convenience sampling Snowball sampling Stratified sampling

Stratified sampling. This form of sampling is used to ensure that there are sufficient cases representing different values of a variable (in this case ethnic groups). Simple random sampling (answer #1) is not correct because simple random sampling refers to the random selection of a predetermined number of cases from a sampling frame, where each case in the sampling frame has an equal probability of being selected for inclusion in the sample. Convenience sampling (answer #2) is not correct because convenience sampling refers to cases that are selected for inclusion in a study sample because they can be readily accessed by the researcher. Snowball sampling (answer #3) is not correct because snowball sampling is a sampling strategy that involves compiling the sample as the study progresses.

3. The purpose of feedback in a social worker interview is to: a. allow the social worker to confront distortions in the client's thinking b. summarize main points of the interview c. let the client know social worker understands the issue d. give the social worker opportunity to self-disclose for benefit of the client

c. let the client know social worker understands the issue

2. Willard, age 13, comes from a family well known in the community for its members' involvement with drugs and alcohol. Willard's two brothers and his mother, though employed in marginal jobs have serious drug and alcohol problems. His father died from cirrhosis of the liver associated with massive alcohol abuse. Willard is in school, seems well cared for, and has been doing well, but he has recently started to cut classes. In planning a social work intervention with Willard, the social worker will need to consider A) How to help Willard's mother encourage him to continue to work hard. B) How to intervene with the entire family to help them manage their substance problems. C) That family substance abuse is a major factor in becoming a substance abuser and that Willard will need special help to avoid following in the family path. D) That an abuse report should be filed.

That family substance abuse is a major factor in becoming a substance abuser and that Willard will need special help to avoid following in the family path.

When writing about the duality of the "Black experience" and the dual perspective, Leon Chestang identified two systems. Which of the following is the larger, more dominant system that provides instrumental and pragmatic needs for physical survival and quality of life? The Structural System The Nurturing System The Sustaining System The System of Basic Needs

The Sustaining System. Leon Chestang identified two systems: The Nurturing System and the Sustaining System. The Sustaining System is the larger, more dominant system that provides instrumental and pragmatic needs for physical survival and quality of life, such as political power, shelter, and employment. The Nurturing System (answer #2) is an incorrect answer to this question, as it relates to the person's immediate family and community environment. The other two answer choices (answers #1 and #4) are incorrect, as they are not real terms related to the information provided in the question.

In a Task-Centered Treatment approach, once a social worker establishes the client(s) needs, which of the following is the BEST option? As the expert, the social worker should be the primary person to establish the treatment goals The client should be the primary person to establish the treatment goals The client and the social worker should jointly establish the treatment goals A medical doctor should be included in establishing the treatment goals

The client and the social worker should jointly establish the treatment goals

A politician recently admitted his arrangement with the pharmaceutical industry, which involved writing key legislation to mandate vaccinations if the industry would, in return, finance the majority of his re-election campaign. According to Kohlberg's Theory of Moral Development, at which level of moral development was the politician operating? Pre-conventional Morality Conventional Morality Post-Conventional Morality Morality of Social Contract

The correct answer is #1 - Pre-conventional Morality. Pre-conventional Morality is Level 1, Stage 2 and involves "you scratch my back and I'll scratch your back" types of behavior. Conventional Morality (answer #2) and Post-Conventional Morality (answer #3) are both incorrect, as they are both identified as levels involving moral reasoning that goes beyond a self-focused perspective. Morality of Social Contract (answer #4) is not correct because it is a stage, not a level, according to Kohlberg. Moreover, it involves higher moral reasoning than that demonstrated by the politician.

A 22-year-old female client is considering having an abortion. Her social worker is pro-choice and feels passionately about the issue. Which of the following is the FIRST thing that the social worker should do in this situation? She should talk with the client about abortion alternatives She should refer the client to another social worker She should help the client obtain the services she needs to get an abortion She should ask the client to share her rationale for choosing abortion

The correct answer is #4 - She should ask the client to share her rationale for choosing abortion. Keep in mind that a social worker should always seek to understand. Talking with the client about abortion alternatives (answer #1) and referring the client to another social worker (answer #2) are both not correct because once the client has provided more information, the social worker can ascertain if the client lacks information about other options or needs to be referred to another social worker. Helping the client obtain the services she needs to get an abortion (answer #3) is not correct because if the social worker helps the client obtain abortion services, she may miss an important opportunity to gain further understanding of the client's circumstances and alternatives. Remember, the question asks for the FIRST thing the social worker should do in this situation.

A social worker has completed an assessment of a client and is in the process of recommending a placement for a client's treatment. Which of the following is the FIRST consideration in selecting a treatment approach for the client? The nature of the client's problem(s) The level of care needed The treatment provider's knowledge and skill level The philosophy of the agency

The level of care needed. While all of the other answer options: the nature of the client's problem(s) (answer #1), the treatment provider's knowledge and skill level (answer #3), and the philosophy of the agency (answer #4) are important, the level of care needed must be the first consideration. If the client needs hospitalization, incarceration, home healthcare, or hospice, the treatment options would be much different than if the client can receive services on an out-patient basis.

2. When reviewing a social worker's performance, the supervisor notes that the social worker conveys little empathy toward clients who have recently left welfare and holding first jobs. In order to help the social worker develop empathy with her clients, the supervisor should: a. explain welfare-to-work procedures from the client's perspective b. suggest the social worker enter therapy to become a more empathetic person c. model empathetic communication when engaging with the social worker d. clarify the agency's reason for supporting these clients

c. model empathetic communication when engaging with the social worker

8. A client with a history of drug abuse was referred to a social worker by a concerned relative. The social worker should FIRST: a. conduct a family interview b. begin treatment, focusing on the reason for the drug abuse c. refer the client for substance abuse treatment d. evaluate the client's motivation for change

c. refer the client for substance abuse treatment

A social worker has been working with a 35-year-old divorced woman who has joint custody of her son, but is the custodial parent. Her ex-husband, whom the social worker has never met, is taking the client to court in an attempt to gain full custody of the son. The social worker receives a subpoena from the husband's attorney requesting records regarding the wife's therapy sessions with the social worker. The social worker is required to do which of the following? The social worker must release the entire record of the therapy sessions to the attorney The social worker is allowed to release only those records that make direct reference to the wife's relationship with the son, not her treatment issues The social worker is required to respond to the subpoena, but is not required to release records since the subpoena was issued by the attorney and not a judge The social worker does not have to respond to the subpoena because the client's records are confidential

The social worker is required to respond to the subpoena, but is not required to release records since the subpoena was issued by the attorney and not a judge

A client asks the social worker to view his therapy records. The social worker knows that showing the records to the client will be harmful given the client's profile. What is the social workers ethical responsibility? The social worker should show the client his or her records as the client has the right to view them The social worker should limit the client's access to his records and provide assistance in interpreting the records shown to the client The social worker should deny the client access and explain the reasons for doing so The social worker should consult with a colleague

The social worker should limit the client's access to his records and provide assistance in interpreting the records shown to the client

6. Social workers who work with cancer patient support groups use flexible contracts because A) The treatment circumstances of each member vary and influence both attendance and level of participation. B) Clearly established agreements raise issues of authority and control which interfere with a support group's autonomy. C) Support groups are not effective when contracting is defined and shaped by sponsoring agencies. D) Contracts that are flexible make it possible to compose groups of patients with a wider diversity of issues which strengthen mutual aid prosperities in the group.

The treatment circumstances of each member varies and influences both attendance and level of participation.

As the toddler reaches two years of age, the posture of the child will grow more erect, and the brain will reach about 4/5's of its final size. The trend of slower growth will continue as the child continues to age at the stage of 3 years old. The head will continue to experience slow growth until about age 5, where the child's head will be almost as large as that of an adult.

Throughout these years, the child's body will become more adult like in proportion, and at age five should resemble that of an adults though on a smaller scale.

A social worker is seeing a new client, who is a college student, for therapy. After speaking with the client about her current enrollment at the university, the social worker spent a significant amount of time urging the client to work less so she could focus on school. The social worker wants the client to get all "A" grades this semester because she reminds him of his daughter at that same stage in life, as well as the anxieties that he experienced during that time in his life. The client really wants to please the social worker because he reminds her of her dad. Which of the following is reflected in the client? Transference Counter-transference Projection A dual relationship

Tranference. In a therapeutic context, transference refers to redirection of a client's feelings from a significant person to the social worker. Counter-transference (answer #2) is incorrect because this is the redirection of a social worker's feelings toward a client, or more generally speaking a social worker's emotional entanglement with a client, which was reflected in the first part of the question; however, this does not describe what is being reflected in this client. Projection (answer #3) is demonstrated when a person's own unacceptable or threatening feelings are repressed and then attributed to someone else, which is not occurring in this case example. A dual relationship (answer #4) is incorrect, as this refers to a situation where multiple roles exist (not just perceived) between a social worker and a client, such as an actual father and daughter relationship between a social worker and client.

when parents pull children closer to one side

Triangulation

When family identifications are fused

Undifferentiated family ego mass

What should an infant be doing at 6-8 weeks

Vocalize with various sounds

What should an infant be doing at 18 months old?

Walking without supervision/support

When providing information about a client's family background on an assessment, which of the following should be specified: a. Just the relationships of all those living in the client's household (i.e., mother, sister, and grandmother. b. The ages of all those living in the client's household c. Only whether or not there are others living in the client's household d. The names and relationships of all those living in the client's household.

When providing information about a client's family background on an assessment, which of the following should be specified: d. The names and relationships of all those living in the client's household.

When responding to a client's request for information about the disclosure of his/her protected health information, which of the following is NOT required? a. A description of what information was sent b. The purpose of the disclosure c. Disclosures for treatment, payment, or health care operations d. The dates of disclosure and to whom the information was sent.

When responding to a client's request for information about the disclosure of his/her protected health information, which of the following is NOT required? c. Disclosures for treatment, payment, or health care operations

A 32-year-old man is referred to a social worker after his children were removed by child protective services due to allegations of neglect. He tells the social worker that he is not sure that he wants to fight to get them back. He states he doesn't think he wants to go through all the things child protective services wants him to do only to not regain custody in the end. Which response is the best thing the social worker should say to the client? a. "You are free to choose whether or not you want to try to regain custody of the children." b. "It's important to look at how not attempting to get custody of the children will be viewed by child protective services." c. "Your children will benefit from knowing that you at least tried to regain custody, even if you aren't successful." d. "It may be helpful to work toward regaining custody initially, and if at any time you decide you want to to give up, you can do so."

a. "You are free to choose whether or not you want to try to regain custody of the children." ---Insisting the client work to regain custody does not support the client's right to self-determination.

A couple requests help in improving their seven-year marriage. When you ask them about their goals, they begin to argue almost immediately. The husband states his wife nags him too much about chores. The wife states her husband is lazy. How should you proceed with establishing treatment goals? a. Assist them in establishing treatment goals to help them learn to improve their communication and resolve conflict. b. Assist them in setting a goal to help them resolve their arguments around household responsibilities. c. Recommend they receive individual treatment to help them work on individual issues. d. Recommend that they each attend sessions with you individually to set separate goals to work on until they are ready to work together in a session.

a. Assist them in establishing treatment goals to help them learn to improve their communication and resolve conflict. ---It is correct to help them learn new skills that they can use to address the problem of disagreements over chores as well as other problems in their relationship.

10. What is the first thing to assess in on-site basic crisis intervention, such as in the aftermath of a natural disaster? a. Immediate needs for physical safety, food and clothing b. Trauma response c. Future mental health risk d. Needs for various concrete services (e.g., transportation, alternate housing)

a. Immediate needs for physical safety, food and clothing

2. It is necessary to obtain informed consent prior to giving out data about a patient. Which of the following statements is MOST correct regarding this issue? a. You must obtain informed consent from your patient in most situations, but not in all situations. b. If a significant other requests information about your patient, you may release it without informed consent. c. If you are going to consult with a colleague, you must have informed consent from your patient. d. You must always have informed consent from your patient.

a. You must obtain informed consent from your patient in most situations, but not in all situations.

4. The concept of gender roles is BEST defined as: a. attitudes and behaviors attached to individuals because of their sex b. an individual's sex orientation and/or preference c. biological characteristics that influence an individual's behavior d. interplay of biological and sociocultural influences

a. attitudes and behaviors attached to individuals because of their sex

9. When conducting an assessment of the risk factors of child abuse for a child of a different ethnic background, a social worker must FIRST: a. be aware of how personal cultural biases affect the social worker's ability to deal with issues of diversity b. realize that assessment models are not a substitute for clinical judgment or experience c. use a strengths' perspective rather than a deficit model to form a partnership with the child's family d. understand the need for appropriate eye contact, tone of voice and question techniques during the assessment

a. be aware of how personal cultural biases affect the social worker's ability to deal with issues of diversity

9. In adult survivors of childhood sexual abuse, the most frequently encountered defense mechanism is: a. denial b. intellectualism c. suppression d. projection

a. denial

"I Can't fight it - may as well get ready for it" - stop fighting death - accepts fate

acceptance - final stage of grief and dying

amphetamine (adderall)

adhd meds

dextroamphetamine (dexadrine, dextrostat)

adhd meds

methylphenidate (ritalin, metadate, concerta, daytrana)

adhd meds

child being unable to use a caregiver as secure base and has inability to explore - child distressed upon seperation - shows reluctance toward caregiver upon return - not able to be comforted by stranger - parent has inconsistent response to child distress

ambivalent/resistant

"Who is to blame" - express envy and rage due to hidden feelings of resentment, jealousy, frustration towards others; people in this stage are hold to care for and deal with

anger - second stage of grief and dying

alprazolam (xanax)

anxiety meds

buspirone (buspar)

anxiety meds

clonazepam (klonopin)

anxiety meds

lorazepam (ativan)

anxiety meds

theory focused on internal states, such as motivation, problem solving, decision-making, thinking, attention and how we interact with the world

cognitive theory

effects are major weight gain and change in metabolism, increase risk of diabetes and high cholesterol

atypical antipsychotic meds

little response from child to seperation or reunion, due to child feeling there is not an attachment - no distress upon departure of caregiver - no response to return of caregiver - treats strangers equal to caregiver - parent doesn't respond to child when child is in distress - discourges crying behavior

avoidant

7. A young, beginning social worker has some intrapersonal and interpersonal struggles with women in authority positions that stem from her poor relationship with her mother. The social worker's client is a female 50-ish successful CEO of an Internet start-up. The social worker is likely to struggle in therapy due to: a. Counter-resistance b. Counter-transference c. Transference d. Resistance

b. Counter-transference

4. A man is referred by his primary care provider and presents with a diagnosis of Hypochondriasis. What is the FIRST crucial step in treatment? a. History taking b. Establishment of trust and appreciation for the patient's problems c. Education about the link between stress, emotions and lifestyle and physical health d. Treatment planning

b. Establishment of trust and appreciation for the patient's problems

A 16-year-old client is seeking services from an outpatient social worker after becoming the victim of a sexual assault. The client's mother has signed a general release of information that allows the social worker to talk to the school. The social worker receives a call from the client's math teacher who states the client's grades have declined and she wants to know if there is something "going on" with the client lately. How should the social worker respond? a. Contact a supervisor before disclosing any information. b. Talk to the client about whether or not she would want any information disclosed and the possible implications of disclosing information. c. Contact the client's mother to determine how much information she is comfortable with the teacher knowing. d. Tell the math teacher that the client was sexually assaulted.

b. Talk to the client about whether or not she would want any information disclosed and the possible implications of disclosing information. --It is appropriate to discuss with the client whether or not she wants her teacher to be made aware of the assault, and she should be warned of the possible implications.

- age 7 to 11 - think logically - have difficulty understanding abstract concepts - can reverse logics

concrete operational stage (piaget)

A social worker has started to treat a woman who has been diagnosed with dependent personality disorder. The woman reports a variety of failed relationships. She acknowledges that she often makes unhealthy choices in the kinds of partners that she picks. When establishing a therapeutic relationship with the client, what sort of information should the social worker keep in mind? a. The client is likely to have difficulty expressing her emotions and most likely lacks basic social skills. b. The client may be superficially compliant with treatment to try and gain approval but may struggle to make lasting change. c. Establishing rapport will be difficult and the client is likely to drop out of treatment early. d. The client's dramatic presentation and desire to be the center of attention may interfere with developing the therapeutic relationship.

b. The client may be superficially compliant with treatment to try and gain approval but may struggle to make lasting change. ---Clients with dependent personality disorder typically need approval and reassurance, so they often try to appear cooperative to please the social worker.

A family has adopted a seven-year-old boy who had previously experienced severe neglect. The adoptive parents are meeting with a social worker because they are frustrated that he continues to hoard food. They report that they often find food under his bed and he "steals" extra food to put in his backpack for school. What recommendations should the social worker make? a. They should install locks on the cupboards and the refrigerator so they can better monitor his food intake. b. They can provide the child with a food basket where he can keep his healthy snacks and they can refill the basket when it is nearing empty. c. The parents should ignore the behavior as it will likely go away on its own once the child realizes that the family has plenty of food. d. The parents should take away privileges each time they discover that he is hoarding food.

b. They can provide the child with a food basket where he can keep his healthy snacks and they can refill the basket when it is nearing empty.

9. Professional ethics and professional values differ in which of the following: a. Values are a subset of ethics. b. Values deal with what is right or wrong in practice, and ethics deals with the principles of practice. c. Values and ethics are basically the same and are evaluated based on the specific situation. d. When a social worker makes a judgment that is ethical, you can assume it is a judgment that also upholds professional values.

b. Values deal with what is right or wrong in practice, and ethics deals with the principles of practice.

1. A fifteen year old girl has been sent home from school on three occasions for drinking alcohol at school. Each time, the mother becomes angry and threatens to punish the girl but is convinced by the father to give her another chance. Which of the following BEST characterizes the father's behavior in this situation? a. triangulation b. enabling c. advocating d. manipulating

b. enabling

1. The phase of the helping process which leads to subsequent implementation of intervention is: a. exploration and summation b. exploration, assessment and planning c. initiation of established rapport d. clarification and analysis

b. exploration, assessment and planning

6. When interviewing a client whose speech patterns are different from the social worker's, the social worker's PRIMARY consideration should be that: a. communication patterns vary due to economic conditions b. speech is influenced by culture and experience c. psychosocial conditions will cause variations in speech d. speech is influenced by group taboos and norms and may indicate false beliefs

b. speech is influenced by culture and experience

"lets strike a deal" - people in this stage are hoping to find a way to postpone death

bargaining - third stage of grief and dying

effects of stress, violence, and trauma?

- regression / hault of brain development - inability to identify, express, manage emotions - dissociation - mentally seperate from experiences - difficulty self-regulating behaviors - difficulties thinking clearly, reasoning, problem solving

theories that support individual's cognitive functioning and learn through acting on their environment

behavior theory / cognitive theory / learning theory

3. A soldier with combat-related Posttraumatic Stress Disorder desires to return to combat. What is the first thing you should do? a. Allow the soldier full self-determination, returning when he deems appropriate. b. Inform the client that return will likely cause an increase in symptoms and is not advised. c. Contact the commanding officer to determine if this is an option for the soldier. d. Assess progress in treatment and current level of functioning.

d. Assess progress in treatment and current level of functioning.

8. When treating elder adults with depression, the BEST approach to treatment is: a. Cognitive behavioral therapy coupled with family interventions. b. Significant reliance on psychiatry because medication management is most important. c. Treatment of elder adults is similar to other adults, so no significant alterations are needed. d. Coordination between medical and mental health due to the influence of medical conditions on psychiatric symptomatology.

d. Coordination between medical and mental health due to the influence of medical conditions on psychiatric symptomatology.

A social worker is employed at a doctor's office. The program director, who is a doctor at the office, reports that the office will begin providing services via video conferencing to established clients who live in rural areas. The purpose is to increase access to services for clients who lack reliable transportation. What should the social worker do? a. Begin services with clients who have already begun receiving service via video conferencing with a doctor. b. Explain to the program manager that social workers cannot establish therapeutic relationships with clients when clients are not physically present. c. Inform the program manager that social workers are not allowed to provide telemedicine. d. Discuss with each client the potential risks, benefits, and consequences of receiving services via electronic media.

d. Discuss with each client the potential risks, benefits, and consequences of receiving services via electronic media. ---It is important to discuss the risks and potential consequences with clients so that clients are aware how services delivered via electronic media differ from face-to-face contact.

A social worker is meeting with a 50-year-old African-American male who acknowledges he has some suicidal ideation. He was previously diagnosed with major depression and was recently discharged from an inpatient hospitalization. He reports that following his hospitalization he has more energy and feels better. He lives at home with his wife and his adult children have moved out of the home. He works in construction and states that he works about 60 hours per week. What suicide risk factors should the social worker pay close attention to? a. The long hours that he works put him at a higher risk of suicide. b. The client is African-American and therefore, statistically, a higher suicide risk. c. Having adult children who have moved out of the home puts him at a higher risk. d. His improvement during his hospitalization may actually give him the energy to complete suicide.

d. His improvement during his hospitalization may actually give him the energy to complete suicide. ---His improvement in the hospital puts him at a higher risk because he may now have the energy to plan and follow through with suicide.

A social worker is working on values clarification with a client who has a dependence on alcohol. The social worker is also assisting the client in recognizing the consequences and the impact his behaviors have on others. The social worker uses a readiness ruler to help assess the client's readiness to change. These methods are most likely to be effective when the client is in which stage of the Transtheoretical Model? a. Action b. Maintenance c. Preparation d. Precontemplation

d. Precontemplation --All these strategies are most likely to be effective with a client who is precontemplative.

5. Client: "I'm depressed. I've lost my hair at a much quicker rate than I thought possible. I'm terribly embarrassed to go out in public. I'm feeling extremely anxious and uneasy about my baldness." Social Worker: "You're anxious and uneasy about the transformations that are a natural process of aging." The social worker's response is: a. Encouragement b. Summarization c. Paraphrase d. Reflection of feeling

d. Reflection of feeling

2. Parents continually refuse to communicate with their child who is in a residential treatment facility. The social worker should FIRST: a. respect the parent's need for separation by not contacting them b. request that the child be discharged c. contact the parents about the child's placement d. assess the child's response to the parents' actions

d. assess the child's response to the parents' actions

8. Which of the following approaches would be MOST appropriate to use in evaluating client satisfaction with the social worker's provision of services? a. using a single system design, evaluate the quality of services b. in the final interview, ask the client about their level of satisfaction c. after submitting process recordings, ask supervisor for feedback d. at termination, ask clients to complete a service evaluation and satisfaction form

d. at termination, ask clients to complete a service evaluation and satisfaction form

10. Which of the following approaches would be MOST appropriate to use in evaluating client satisfaction with the social worker's provision of services? a. using a single system design, evaluating the quality of services b. in the final interview, ask the clients about their level of satisfaction c. through process recordings, ask the supervisor for feedback d. at termination, ask clients to complete a services evaluation and satisfaction form

d. at termination, ask clients to complete a services evaluation and satisfaction form

4. A social worker is leading a women's group with the purpose of improving personal and environmental circumstances. The social worker asks the members to identify the area in which they feel most competent. The social worker is following an intervention strategy BEST described as: a. reinforcement b. endorsement c. enhancement d. empowerment

d. empowerment

5. The BEST example of the educational component of supervision is: a. monitoring the caseload b. providing in-service training c. utilizing individual and group conferences d. refining knowledge and skills

d. refining knowledge and skills

10. A social worker is receiving a client with long-standing depression. It becomes known that the client has racist beliefs. The social worker should: a. respectfully confront the client's prejudiced beliefs b. educate the client about the negative effects of racism c. refer the client to another therapist who might have similar belief d. respect the client's beliefs and focus on the presenting problem

d. respect the client's beliefs and focus on the presenting problem

3. Common indicators of sexual, physical and emotional abuse include all of the following EXCEPT: a. violence against younger children b. sexualized behavior c. isolation d. stuttering

d. stuttering

5. Which type of interviewing question is MOST likely to generate client defensiveness? a. open ended b. closed c. leading d. why

d. why

Refusing to acknowledge or recognize the reality and implications of painful, anxiety-provoking experiences.

denial

"this isn't really happening to me?" or "I am fine, there is nothing wrong" - usually short lived

denial - first stage of grief and dying

"I am going to die, what's the point" - emotionally sensitive, uncomfortable, fearful, sense of tremendous loss; don't try to cheer them up - let them go through emotions

depression - fourth stage of grief and dying

bupropion (wellbrutin)

depression meds

citalopram (celexa)

depression meds

escitalopram (lexapro)

depression meds

fluoxetine (prozac)

depression meds; bipolar

paroxetine (paxil)

depression meds; bipolar

sertraline (zoloft)

depression meds; bipolar

Shifiting repressed feelings from where they originate to some other object.

displacement

repeated negative acts or statements directed at child, exposure to repeated violent, brutal, intimidating acts or statements among members of household, cruel actions to attempt to gain submission, enforce maximum control, modify child's bx, rejection of child

emotional abuse and maltreatment

parents pour emotional problems on to child

family projection process

- age 12 to adult - think abstractly, logical thought, deductive reasoning, systematic planning - can understand hypothetical questions

formal operational stage (piaget)

a set of social and behavioral norms that are widely considered to be appropriate for individuals of a specific sex

gender role

Form of denial in which object of attention is presented as all good masking true negative feelings toward the other

idealization

The unconscious modeling of one's self upon another person's behavior.

identification

Rationalizing and making generalizations about anxiety provoking issues to minimize pain and anxiery.

intellectualization

Identifying with some idea or object so deeply that it becomes a part of that person.

introjection

refocusing of aggression or emotions evoked from an external force into one's self

inversion

Inability to simultaneously experience the cognitive and affective components of a situation.

isolation

- success and failure is related to sense of contribution to a new environment

margin/matter community theory

age 40 to 60 - realize you will not live forever creates tension - try to cling to youth, fail in process of self-realization - "confront one's shadow" - religiosity may increase

middle life stage (carl jung)

family projection process repeats itself from each generation

multigenerational transmission process

failure to provide and maintain adequate food, clothing, medical care, supervision, education

neglect

believing something exists even when you cannot see it

object permanence

age 60 and over - consciousness is reduced - jung thought death is ultimate goal of life - won't face death with fear but hope for rebirth

old age stage (carl jung)

7-year-old child is refusing to sleep in his own bed at night. When the parents sit with him in his room, he is fine. When they leave the room, he begins to cry. When they come back in, he stops crying. This is an example of Operant conditioning Classical conditioning Punishment cycles Higher order conditioning

operant conditioning

peace keeper role, mediating and reducing tension between conflicting parents; may be in response to unconscious anxiety about family breakdown

peace keeper

head injuries, bruises, cuts, lacerations, internal injuries, burns, scald, reddening, blistering of tissues by any substance or force, injuries to body in any way, death

physical abuse

- age 2 to 7 - language development - no concrete logic, not able to manipulate mental thoughts, not able to "put themselves in others shoes" - use symbols and role play

pre-operational stage (piaget)

believe healing occurs through attention to transferences and treatment relationships

psychodynamic theory

theory that believes early childhood experiences are central in patterning an individual's emotions which leads to problems in life

psychodynamic theory

theory that believes unconscious and conscious mental activity serves as motivating force in human bx

psychodynamic theory

theory that is concerned with how internal processes such as needs, desires, emotions motivate human bx

psychodynamic theory

theory that says individual's get overwhelmed with internal/external demands and use ego defense mechanisms

psychodynamic theory

losing of same aspect of development already achieved due to undue anxiety causing a person to revert to a previously attained stage or lower level of adaptation

regression

Pusing a negative or painful image, thought, or idea out of consciousness to avoid the associated pain. This is the primary defense mechanism.

repression

a visible symptom of troubled family system

scapegoat

Chlorpromazine (thorazine)

schizophrenic drug

Fluphenazine

schizophrenic drug

Perphenazine

schizophrenic drug

haloperidol (haldol)

schizophrenic drug

paliperidone (invega)

schizophrenic drug

quetiapine (seroquel)

schizophrenic drug

Clozapine (clozaril)

schizophrenic drug but can really affect white blood cell counts

aripiprazole (abilify)

schizophrenic drug; bipolar

lurasidone (latuda)

schizophrenic drug; bipolar

olanzapine (zyprexa)

schizophrenic drug; bipolar

risperidone (risperdal)

schizophrenic drug; bipolar

ziprasidone (geodon)

schizophrenic drug; bipolar

secure base where child sees caregiver as secure base from which to explore the world - protest caregiver departure - seek proximity and comfort upon caregiver's return -- show preference for caregiver over a stranger

secure

- birth to 2 years old - perception of world is limited to sensory and motor - learn through sucking, grasping, listening - need to learn object permenance

sensorimotor stage (Piaget)

rape, intercorse, sodomy, fondling, oral sex, incest, sexual penetration, sexual exploitation

sexual abuse

people tend to have many similarities with those sharing birth order

sibling position

What should an infant be doing at 4-6 weeks

smiling at parent

manifestation of emotional anxiety into physical symptoms

somatization

Repressing, dissociating or disconnecting important feeling that are dangerous to psychic well-being. Causes person to get out of touch with her/his feelings and feeling to "fragmented self."

spitting

person replaces 1 feeling or emotion in place of another

substitution

Way knowledge and skills can be adapted if they are to be made relevant and applicable to different situations encountered in social work practice

transferability

- puberty until 35 - 40 - maturing sexuality, growing consciousness, realization that carefree days of childhood are gone - strive to gain independence, find a mate, raise family

youth stage (carl jung)


Ensembles d'études connexes

ExamFX Field Underwriting Procedures

View Set

Social Psychology Exam #3, Chapter #7

View Set

Programing Chapter 10 Quiz Characters, Strings, and string Class

View Set

What is the supreme law of the land?

View Set